Você está na página 1de 119

CLCULO DIFERENCIAL

FUNCIONES CONTINUAS

PARA LOS ESTUDIANTES DE CIENCIAS

CLCULO DIFERENCIAL:
FUNCIONES CONTINUAS
Primera Edicin
Agosto del 2013
AUTOR/EDITOR: Gustavo, Marca Castromonte
Tipeado por el autor en LATEX
e-mail: marcagustavo@yahoo.com

Este material puede ser compartido, reproducido en cualquiera de sus formas, con total
libertad, para fines acadmicos, de manera muy especial por los estudiantes de la Facultad de
Ciencias de la Universidad Nacional de Ingeniera.

Hecho el Depsito Legal en la Biblioteca Nacional del Per


NMERO DEL DEPSITO LEGAL:

Lima - Per

...................

Prefacio

iii

Agradecimientos

ndice general
Prefacio

iii

Agradecimientos

ndice general

vii

1 Continuidad de Funciones

1.1. Primeras Definiciones . . . . . . . . . . . . . . . . . . . . . . . . . . . . . .

1.2. lgebra de Funciones Continuas . . . . . . . . . . . . . . . . . . . . . . . . 14


1.3. Continuidad y las Desigualdades

. . . . . . . . . . . . . . . . . . . . . . . 24

1.4. Continuidad y la Composicin de Funciones . . . . . . . . . . . . . . . . . 27


1.5. Continuidad y las Sucesiones

. . . . . . . . . . . . . . . . . . . . . . . . . 34

1.6. Continuidad y el Valor Absoluto . . . . . . . . . . . . . . . . . . . . . . . . 43


1.7. Continuidad y las Funciones Trigonomtricas . . . . . . . . . . . . . . . . . 51
2 Continuidad Uniforme

53

2.1. Uniformemente Continua y la Sucesin de Cauchy . . . . . . . . . . . . . . 58


2.2. Uniformemente continua y los conjuntos acotados . . . . . . . . . . . . . . 59
2.3. Adicin de Funciones Uniformemente Continuas . . . . . . . . . . . . . . . 60
2.4. Sustracin de Funciones Uniformemente Continuas . . . . . . . . . . . . . 62
2.5. Multiplicacin Escalar y Uniformemente Continua . . . . . . . . . . . . . . 63
3 Teoremas Fundamentales de Continuidad

65

3.1. Continuidad y los Intervalos . . . . . . . . . . . . . . . . . . . . . . . . . . 73


vii

viii
4 Problemas Resueltos
5 Problemas Propuestos

NDICE GENERAL
81
109

...........

1
Continuidad de Funciones
1.1.

Primeras Definiciones

Dentro del estudio de la asignatura del Clculo Diferencial la primera parte est dada
al estudio de: Los nmeros naturales, enteros, racionales y reales, teniendo como esencia a
los nmeros reales como cuerpos ordenados completos y el tema de funciones, como caso
especial las funciones reales.
En esta segunda parte, los temas fundamentales competen a: El estudio de los lmites,
continuidad y derivabilidad de funciones reales de variable real.
El presente material tiene como objetivo detallar la parte terica y prctica del estudio
de las funciones continuas, con la cual se pretende el desarrollo matemtico en el estudiante, mediante la reflexin de sus conceptos, propiedades, teoremas, y las conexiones con
los temas pasados como son: las sucesiones y los lmites.
Esperando que sea de su agrado, y que colabore en la formacin de cada uno de los
estudiantes que se acercan a esta obra, iniciamos con la definicin fundamental, que da
origen a todo este gran captulo, de funciones continuas en un punto.
Definicin 1.1 (Continuidad de una Funcin en un Punto). Sea f : X R una funcin
con a X. Diremos que f es una funcin continua o simplemente continua en el punto
x = a si y solo si para cada  > 0, existe > 0 de modo que:x X y |x a| < , implica
que |f (x) f (a)| < .
El estudiante notar que no hay necesidad que el punto a, sea un punto de acumulacin del dominio de la funcin.
3

CAPTULO 1. CONTINUIDAD DE FUNCIONES

Tambin, nicamente podemos hablar de continuidad para puntos del dominio de la funcin. La definicin se parece mucho a la definicin de lmite de una funcin, pero si se
mira con ms detenimiento, nos daremos cuenta que no lo es.
Podemos luego tener con respecto al punto a, dos casos:
Caso 1 El punto a es un punto de acumulacin respecto al dominio de la funcin f , la
cual es X.
Caso 2 El punto a no es un punto de acumulacin, en otras palabras, es un punto aislado
respecto al conjunto X.
A continuacin se da un ejemplo para explicar el caso en que el punto de referencia sea
aislado.
Ejemplo 1.1. Considere f : (1, 2) {5} R una funcin definida por f (x) = x2 . Pruebe
que f es continua en x = 5.
Prueba: Sea  > 0 arbitrariamente tomado, basta ahora considerar = 1, veamos que
este valor es el adecuado. Sea x Dom(f ) y |x 5| < 1, por lo tanto 1 < x 5 < 1,
entonces 4 < x < 6, es decir el nico valor del dominio que verifique ello es x = 5, luego
calculamos
|f (x) f (5)| = |f (5) f (5)| = 0 < 
basta con ello para haber demostrado lo pedido.
Note que para este caso se ha tomado = 1 se pudo haber considera = 2, la idea es
que la vecindad alrededor del 5, no tome puntos del intervalo (1, 2).
Ejemplo 1.2. Sea f : { n1 : n {1, 2, 3, 4, 5, 6, 7, 8}} R una funcin definida . Pruebe
que f es continua en x = 15 .
Prueba: Detalle el estudiante.
De manera similar, si tenemos f : {1, 2, 3, 4, 5, 6, 7, 8} R una funcin, a pesar que se
desconoce su regla de correspondencia, podemos asegurar que f es una funcin continua
en x = 1, tambin en x = 2, del mismo modo hasta x = 8, es claro que f no es continua
en x = 0, ya que no se encuentra en su dominio.

1.1. PRIMERAS DEFINICIONES

Un poco ms fino la expresin, podemos tener la funcin f : { n1 : n N} R la cual


es una funcin continua en cada punto de su dominio, se demuestra que cada punto de su
dominio es un punto aislado. A continuacin definamos lo que es un punto aislado.
Definicin 1.2 (Punto aislado). Sea A R no vaco y x A. Diremos que x es un punto
aislado del conjunto A si y solo si existe > 0 de modo que hx , x + i (A \ {x}) = .
Por ejemplo, si consideramos el conjunto A = h1, 2i {1, 0, 2, 3}, se tiene que x = 0
es un punto aislado de A y x = 1 no es un punto aislado del conjunto A.
Recordar tambin que si todo punto de un conjunto es aislado, diremos que dicho
conjunto es discreto.
Observacin: Si consideramos la funcin f (x) = sen( x1 ) no es continua en x = 0,
es decir es discontinua en 0, ya que 0 6 Dom(f ). Luego si el punto no se encuentra en
el dominio de la funcin a trabajar, no tiene sentido hablar de continua en dicho punto.
Aqu la palabra discontinua es la negacin de la continuidad.
Los caso dados anteriormente, son especiales, pero el trabajo en esta parte es entender
la relacin entre los radios  y , para ello lo detallaremos mediante una serie de ejercicios
significativos.
Ejemplo 1.3. Considere la funcin constante f (x) = 3. Demostrar que f es continua en
todo punto de su dominio.
Prueba: Sea a Dom(f ) cualquiera, ahora consideremos  > arbitrario, podemos tomar
luego = 1 y sea
x Dom(f ) = R |x a| < = 1
es decir x R y a 1 < x < a + 1
Para finalizar calculemos para los valores de x en la inecuacin anterior
|f (x) f (a)| = |3 3| = 0 < .

Ejemplo 1.4. Sea f (x) = 8 una funcin constante con dominio en todo R. Demuestre la
continuidad de f en el punto x = 7.

CAPTULO 1. CONTINUIDAD DE FUNCIONES

Prueba: Detallar, el proceso de la demostracin.


Note que se ha considerado para este caso = 1, pero es claro, que se puede tomar
tambin = 2. En general, para este ejercicio, es posible tomar cualquier > 0 y sigue
cumpliendo la continuidad. Veamos otro ejemplo, donde se realice otra tcnica para su
clculo.
Ejemplo 1.5. Considere la funcin f (x) = 2x, x R. Demuestre que f es continua en
x = 6.
Prueba: Sea  > 0 tomado arbitrariamente, ahora tomemos = 2 >0, veamos si con este

radio cumple con la definicin de continuidad. Sea x Dom(f ) = R y |x 6| < , luego
2
podemos calcular

|f (x) f (6)| = |2x 12| = 2|x 6| < 2 = .
2

Note que para este caso se ha considerado la mitad del radio  como el adecuado,
para que las ecuaciones puedan salir en forma adecuada. Veamos otro ejemplo.
Ejemplo 1.6. Sea f (x) = x + 5, x R. Demostrar que f es una funcin continua en 10.
Prueba: Damos un intento:
|f (x) f (10)| = |x + 5 15| = |x 10| < , luego segn la expresin |x 10| <  tenemos
que considerar = .
Formalmente:
Sea  > 0 arbitrario, ahora tomemos =  > 0 y para cada
(1.1)

x R |x 10| < = 
Calculamos en la parte final de la demostracin
1.1

|f (x) f (10)| = |x + 5 15| = |x 10| < = 

1.1. PRIMERAS DEFINICIONES

Observacin: El estudiante notar que el proceso demostrativo se ha realizado en


dos pasos, un primer paso es el intento, en la cual nos daremos cuenta del adecuado a
utilizar.
Mientras que en el segundo paso, tenemos la situacin formal, en la cual desarrolla la
defincin en etapas, pero con el valor adecuado descubierto en la parte primera. Veamos
otro ejemplo.
Ejemplo 1.7. Sea la funcin g(x) = 5x+8, x R. Pruebe que g es una funcin continua
en 2.
Prueba: Intentamos:
Calculemos el siguiente valor absoluto
|g(x) g(2)| = |5x + 8 18| = |5x 10| = 5|x 2| < 
por lo tanto, observando la desigualdad anterior 5|x 2| < , tenemos |x 2| < 5 , luego
podemos considerar = 5 .
Sea  > 0 un real positivo cualquiera, luego consideremos = 5 .
Veamos la parte formal: Sea x Dom(g) = R y |x 2| < = 5 , luego calculemos
|g(x) = g(2)| = |5x 10| = 5|x 2| < .

El siguiente ejemplo tomar a la funcin cuadrtica, para iniciar otro estilo de demostracin.
Ejemplo 1.8. Sea f (x) = x2 la funcin cuadrtica. Pruebe que f es continua en 8.
Prueba: Intento:
|f (x) f (8)| = |x2 64| = |x + 8||x 8|

(1.2)

el problema, es el factor |x+8|, la idea es poder acotarlo superiormente, para ello podemos
considerar un inicial, la cual puede ser = 1, luego |x 8| < 1, entonces |x + 8| < 17
luego de la ecuacin (1.2) tenemos
= |x + 8||x 8| < 17|x 8| < 

CAPTULO 1. CONTINUIDAD DE FUNCIONES

por lo tanto |x 8| <


,
17

de este modo, tenemos = mn{1, 17 }.

Veamos la demostracin en formal.


Sea  > 0 cualquiera, consideremos = mn{1, 17 }, luego tenemos
(1.3)

x Dom(f ) = R |x 8| <
de la ecuacin (1.3) se desdobla en dos expresiones
|x 8| < 1 |x 8| < 1 15 < x + 8 < 17
por lo tanto

(1.4)

|x + 8| < 17
ahora de la ecuacin (1.3) tenemos
|x 8| <
|x 8| <


17


17

(1.5)

ahora, para finalizar, calculemos


(1.4)

(1.5)

|f (x) f (8)| = |x2 82 | = |x + 8||x 8| < 17|x 8| < 17


= .
17

Observacin: El estudiante notar la clave de este ejercicio es acotar adecuadamente


el valor absoluto |x + 8|, tomando el mnimo de dos deltas dados o generados.
El el siguiente ejercicio.
Ejemplo 1.9. Sea f (x) = x2 + x, x R. Demostrar la continuidad de f en 2.
Prueba: Primera parte:
|f (x) f (2)| = |x2 + x 6| = |x + 3||x 2|

(1.6)

ya hemos visto que tenemos que acotar superiormente a la expresin |x + 3|, para ello
tomemos un primer delta = 1. Luego tenemos |x 2| < 1, luego 1 < x 2 < 1, es
decir 1 < x < 3, por lo tanto
|x + 3| < 6

(1.7)

1.1. PRIMERAS DEFINICIONES


de las ecuaciones (1.6) y (1.7) tenemos
= |x + 3||x 2| < 6|x 2| < 

luego tenemos |x 2| < 6 , por lo tanto tomando = 6 , basta considerar el mnimo de


estos dos valores = mn{1, 6 }.
Segunda parte:
Sea  > 0 un radio cualquiera o arbitrario, considere = mn{1, 6 } y sea x Dom(R) y
|x 2| < , de aqu de desprende dos casos:
Caso 1:|x 2| < 1, es decir |x 2| < 1, luego 1 < x 2 < 1, entonces 1 < x < 3 es
decir
(1.8)

|x + 3| < 6
Caso 2: Si |x 2| < 6 , esto nos implica
|x 2| <


6

(1.9)

Finalmente podemos calcular


1.8

1.9

|f (x) f (2)| = |x2 + x 6| = |x + 3||x 2| < 6|x 2| < .

Observacin: Es claro que el proceso de factorizacin es clave en este ejercicio.


Ejemplo 1.10. Usando la definicin, pruebe que la funcin f (x) = 2x2 + 3x + 5 es
continua en x0 = 1.
Prueba: En esta parte, procedemos a demostrarlo mediante dos pasos, el primero es la
bsqueda del > 0 adecuado, dado un  > 0 y una segunda donde es el formalismo de la
primera parte.
Primera parte:
Calculemos
|f (x) f (x0 )| = |2x + 1||x + 1|

(1.10)

en la ecuacin (4.1) vemos el primer problema, es distinto si nos hubiera resultado, por
ejemplo 3|x + 1|, ahora para poder transformarlo a esa forma, podemos considerar un

10

CAPTULO 1. CONTINUIDAD DE FUNCIONES

1 =

1
2

> 0 (es un intento), calculemos


|x x0 | = |x + 1| < 1 =

entonces |x + 1| <

1
2

por lo tanto

1
2

1
2

< x + 1 < 12 , entonces


(1.11)

|2x + 1| < 2
listo,juntemos las ecuacin (4.1) y (1.11) por lo tanto
|f (x) f (x0 )| = |2x + 1||x + 1|
< 2|x 1|
< 
es decir
|x + 1| <


2

consideremos, ahora un segundo intento, para ello tomemos 2 = 2 , finalmente podemos


considerar
= min{1 , 2 } > 0
Porqu crees que no podemos considerar el = max{1 , 2 }?

Segunda parte:
Sea  > 0 arbitrario, luego tomando = { 21 , 2 } > 0. Sea x Dom(f ) = R, |x x0 | < ,
entonces |x + 1| < , por lo tanto
|x + 1| <
es decir
|x + 1| <

1
2

|x + 1| <


2

esto no lleva a determinar que

1
2

11

1.1. PRIMERAS DEFINICIONES


finalmente podemos calcular
|f (x) f (x0 )| = |2x2 + 3x + 1|
= |2x + 1||x + 1|
< 2|x + 1|

< 2 =
2
listo, esto nos ha demostrado que f es una funcin continua en 1.

Observacin: Segn el procedimiento anterior, podemos rescatar el hecho que la


demostracin consta de dos partes, en la parte final, se ha formalizado lo que se ha
realizado en la primera.
Tambin ha sido importante conseguir el valor adecuado para el > 0, para nuestro
caso ha sido tomar
= min{1 , 2 }
El estudiante notara tambin que sirve cualquier valor menor positivo que este, es
decir tambin pudo tomarse
0 < < min{1 , 2 }
Veamos ahora otro ejemplo.
Ejemplo 1.11. Pruebe que f (x) = x2 + 3x + 2 es una funcin continua en 2.
Prueba: Se deja al estudiante para su realizacin y aplique la tcnica anteriormente
expuesta.
Tambin puede resolver los siguientes problemas.
Ejemplo 1.12. Pruebe que f (x) = x2 + x + 2 es una funcin continua en 3.
Prueba: Se sugiere realizar la demostracin en dos partes.
Ejemplo 1.13. Pruebe que la funcin f (x) = 4x2 5x 3 es continua en x0 = 2.
Prueba: Hagamos esta demostracin un poco mas ligera, ya no mostraremos la primera
parte, solo el formalismo, pero no debe olvidar que primero hay que descubrir el adecuado.

12

CAPTULO 1. CONTINUIDAD DE FUNCIONES


Sea  > 0 arbitrario o cualquiera, tomemos = min{1, 15 } > 0, ahora sea x

Dom(f ) = R, |x x0 | < , esto quiere decir que


|x 2| < 1 |4x + 3| < 15

(1.12)

tambin
|x x0 | <



|x 2| <
15
15

(1.13)

ahora si, utilizando las ecuaciones (1.12) y (1.13) obtenemos lo siguiente


|f (x) f (x0 )| = |4x2 5x 6| = |4x + 3||x 2| < 14|x 2| < 

Imitar el proceso anterior con el siguiente ejercicio.


Ejemplo 1.14. Pruebe que f (x) = x2 x + 2 es una funcin continua en 4.
Prueba: Se deja al estudiante para su realizacin y aplique la tcnica anteriormente
expuesta.
Ejemplo 1.15. Pruebe que la siguiente funcin f (x) = 2x2 + x + 1 es continua en 1.
Prueba: Imitar el proceso anterior.
Los elementos de aprendizaje son mltiples y muchas veces desconocidos,
pero est perfectamente demostrado que uno de ellos es la repeticin de procesos ya realizados.
Esto quiere decir joven estudiante, que las repeticiones de los procesos aprendidos, constituye una tcnica favorable en el aprendizaje de las matemticas.
Veamos ahora una demostracin de continuidad, pero con otros elementos de trabajo.
Ejemplo 1.16. Supngase que existe k > 0 tal que x, y Dom(f ), |f (x) f (y)|
k|x y|. Demuestre que f es continua en cada punto de su dominio.
Prueba: Sea x0 Dom(f ) arbitrario, veamos que f es continua en x0 , para ello consideremos  > 0 cualquiera, la clave es encontrar un > 0 adecuado, cul podra ser, ese
valor?, intumos que
=


k

13

1.1. PRIMERAS DEFINICIONES

veamos, si es el adecuado, para ello tomemos x Dom(f ), |xx0 | < , luego |xx0 | < k .
Ahora, respecto a ese x calculemos
|f (x) f (x0 )| k|x x0 | < k


=
k

Note la importancia, de haber intudo bien, en el clculo del > 0.


Mas adelante a este tipo de funciones f se les llamar funciones uniformemente continuas.
Aplicar la tcnica anterior al siguiente ejercicio en forma particular.
Ejemplo 1.17. Supngase que existe una funcin f : X R de modo que se cumpla
con x, y X, |f (x) f (y)| 7|x y|. Demuestre que f es continua en cada punto de
su dominio.
Prueba: Detallar su resolucin.
Ejemplo 1.18. Considere una funcin f : R R de modo que se cumpla con x, y
R, |f (x) f (y)| |x y|. Demuestre que f es continua en x = 9.
Prueba: Detallar su resolucin.

Problemas Variados
1. Sea f : R R, de modo que f (x) = 23 . Pruebe que f es continua en 1.
2. Considere la siguiente funcin g(x) = x + 9 con dominio en Q. Pruebe que g es
2
continua en x = .
5
3. Sea f (x) = 5x, x R una funcin. Pruebe que f es continua en 5.

1
,x < 0
4. Se tiene la siguiente funcin f (x) =
Demostrar que f es continua

x + 1 , x 0
en x = 0.

14

CAPTULO 1. CONTINUIDAD DE FUNCIONES


5. Sea f (x) = 3x2 una funcin real con variable con variable en [2, 3]. la funcin f es
continua en 2?, si su respuesta es verdadera, demuestre tal afirmacin, en otro caso
de un contraejemplo.
6. Considere la funcin h(x) =

2 , x < 4

. Demuestre que h no es continua en x = 4.

3 , x = 4
7. Considere f (x) = 2x2 + x 7, con dominio en todo los nmeros reales. Pruebe que
f es continua en 1.
8. Sea f (x) = x2 + x + 10, x [1, 4]. f es continua en 1.
9. Sea f : X R una funcin, donde X es un subconjunto finito de los nmeros reales.
Pruebe que f es continua en cada punto de su dominio.
10. Considere la funcin f (x) =

1.2.

x2 4
.
x+2

Pruebe que f es continua en 4.

lgebra de Funciones Continuas

Adicin de Dos Funciones Continuas


Teorema 1.1. Sean f, g : X R, a X. Si f y g son funciones continuas en x = a,
entonces f + g es continua en x = a.
Prueba: Considere  > 0 en forma arbitraria, debido a la continuidad de las funciones f
y g tenemos
1 > 0 tal que , |x a| < 1 |f (x) f (a)| <


2

(1.14)

tambin tenemos por cierto que


2 > 0 tal que x X, |x a| < 2 |g(x) g(a)| <


2

(1.15)

la clave es tomar = mn{1 , 2 }


|f (x) f (a)| <


2

(1.16)

15

1.2. LGEBRA DE FUNCIONES CONTINUAS

Sea x X, |x a| < |x a| < 1 , por lo tanto se tiene |x a| < 1 , entonces


por la ecuacin (1.14) se tiene
|f (x) f (a)| <


2

(1.17)


2

(1.18)

tambin se cumple con la ecuacin (1.15) que


|g(x) g(a)| <
finalmente podemos calcular
|(f + g)(x) (f + g)(a)| = |f (x) f (a) + g(x) g(a)|
|f (x) f (a)| + |g(x) g(a)|


<
+ =
2 2

Luego de verificar que la suma de dos funciones continuas, es tambin una funcin
continua, podemos hacernos la siguiente pregunta: Si f +g es continua en x = a, entonces
podemos afirmar que f y g son continuas en x = a?
Creemos que en algunos casos se puede cumplir, pero en otros no, luego tenemos que dar
un ejemplo concreto, y al decir concreto, estamos indicando en forma numrica, por lo
tanto podemos considerar una funcin f
f (x) =

1 , x = 0

2 , x 6= 0

tambin la siguiente funcin g


g(x) =

3 , x = 0

2 , x 6= 0

se demuestra que f y g son funciones discontinuas en el punto x = 0, se deja al lector o


estudiante la verificacin de dicha afirmacin, se sugiere realizarlo por definicin, que por
cierto es lo nico que tenemos a la mano, hasta ahora, pero si sumamos ambas funciones,
obtenemos
(f + g)(x) =

4, x = 0

4, x 6= 0

16

CAPTULO 1. CONTINUIDAD DE FUNCIONES

la cual simplemente podemos expresarla por (f + g)(x) = 4, x R, es decir se trata de


una funcin constante, la cual se prueba que es continua en todo punto, luego en x = 0.
Si consideramos la siguiente funcin f (x) = x2 + 2x + 7, es una funcin continua en
2, ya que g(x) = x2 es continua en 2 y h(x) = 2x + 7 lo es tambin en 2 y se cumple
f (x) = g(x) + h(x)
De lo anterior tenemos una forma nueva de demostrar cuando una funcin es continua
en un punto, descomponiendo la funcin por medio de la adicin y que ambas funciones
componentes sean continuas en el mismo punto.
Teorema 1.2. Sean f, g, h : X R funciones continuas en a X. Demuestre que
f + g + h es una funcin continua en el mismo punto a X.
Prueba: Por dato tenemos que f y g son dos funciones continuas, luego por el teorema
1.1 se tiene que f + g es una funcin continua en a, pero como h es continua en a, luego
tambin por el teorema 1.1, tenemos que (f +g)+h es continua en el mismo punto, debemos
recordar que la adicin de funciones continuas goza de la propiedad de asociatividad, es
decir
(f + g) + h = f + g + h
luego tenemos concluido con lo pedido.

Sustraccin de Dos Funciones Continuas


Consideremos ahora el caso acerca de la sustraccin de dos funciones continuas, originando una nueva funcin continua.
Teorema 1.3. Sean f, g : X R, a X. Si f y g son dos funciones continuas en x = a,
entonces f g es continua en x = a.
Prueba: Lo demostraremos por definicin, considere  > 0 dado arbitrariamente,para
poder utilizar la continuidad de f y g al mismo tiempo, podemos considerar un solo > 0
adecuado en vez de dos 1 y 2 . Veamos ello, por continuidad existe un > de modo que:

1.2. LGEBRA DE FUNCIONES CONTINUAS

17

x X con |x a| < , implique que:



2

|g(x) g(a)| <
2

|f (x) f (a)| <

ahora s, consideremos el > 0 como referencia. Sea x X, |x a| < , entonces cumple


con las ecuaciones anteriores, por lo tanto, podemos calcular:
|(f g)(x) (f g)(a)| = |f (x) g(x) (f (a) g(a))|
= |f (x) g(x) + g(a) g(x)|
|f (x) f (a)| + |g(a) g(x)|
= |f (x) f (a)| + |g(x) g(a)|


=
+ =
2 2

Al igual que en el caso de la adicin puede ser que la diferencia f g sea continua
en un punto a, pero que no indica que tanto f como g sean continuas en a. Pero en el
caso que una de ellas sea continua, f o g, la otra obligatoriamente tiene que ser continua,
detallar esta ltima afirmacin.

Ejemplo 1.19. Sea f, g : X R dos funciones. Si f + g y g son dos funciones continuas


en a X. Demuestre que f tambin lo es, en el punto a X.
Prueba: Se deja al estudiante.
Ejemplo 1.20. Dar un contraejemplo de dos funciones f, g : X R, de modo que f g
es continua en a X, pero ninguna de las dos funciones lo es.
Prueba: Se deja al estudiante.
Observacin: Considere la funcin f (x) = sen(x)x2 es una funcin continua en a R,
ya que se trata de la sustraccin de dos funciones g(x) = sen(x), h(x) = x2 las cuales son
continuas en a, respecto a la funcin trigonomtrica sen se trabajar ms adelante.

18

CAPTULO 1. CONTINUIDAD DE FUNCIONES

Multiplicacin de un Escalar por una Funcin Continua


Teorema 1.4. Sea f : X R, a X una funcin continua en x = a, entonces para cada
k R, kf es una funcin continua en x = a.
Prueba: Queremos en esta parte, dividir la demostracin en dos partes:
Si k 6= 0;
Si k = 0.
Veamos el primer caso, considere  > 0 dado arbitrariamente, luego, como f es una funcin
continua en a, debe existir > 0:
Si x X con |x a| < |f (x) f (a)| <


|k|

(1.19)

debe notar el estudiante la importancia del valor absoluto en la ecuacin anterior, podemos
considerar el mismo de la ecuacin (1.19) y sea x X, |x a| < , entonces se tiene
|f (x) f (a)| <


|k|

entonces |k||f (x) f (a)| < , por lo tanto |kf (x) kf (a)| < , finalmente tenemos
|(kf )(x) (kf )(a)| < 

As tenemos la aplicacin siguiente.


Ejemplo 1.21. Demostrar que f (x) = 5x2 , x R es una funcin continua.
Prueba: Esta claro, que se puede demostrar utilizando la defincin, pero ahora tenemos
otro posibilidad, si consideramos g(x) = x2 , por un resultado anterior, g es continua en
cualquier punto de su dominio, luego f (x) = 5g(x) tambin lo es.
Veamos el siguiente ejemplo, donde se tiene que tener cuidado al aplicar la propiedad
anterior.
Ejemplo 1.22. Sabiendo que f (x) = kg(x), con k R y f, g : R R, es una funcin
continua en a. Se puede deducir que g(x) es tambin continua en a?

1.2. LGEBRA DE FUNCIONES CONTINUAS

19

Prueba: Se deja al estudiante, la verificacin, que no siempre es cierto, es decir tenemos


que dar un contraejemplo para su demostracin.
En general podemos tener por verdadero el siguiente resultado.
Ejemplo 1.23. Sean f, g : X R R dos funciones continuas en el punto a X. Si
, R, entonces demuestre que f (x) + g(x) es una funcin continua en a X.
Prueba: Se recomienda al estudiante, realizar la demostracin por medio de la definicin.

Multiplicacin de Dos Funciones Continuas


Siguiendo con este desarrollo de la continuidad ligada a las operaciones fundamentales, llegamos al punto de discutir acerca de la multiplicacin de funciones continuas, es
claro que seguir siendo continua, pero en su demostracin hace uso de una tcnica muy
importante, que todo estudiante que comienza con este buen tema deber advertir.
Teorema 1.5. Sean f, g : X R, a X. Si f y g son dos funciones continuas en x = a,
entonces f g tambin lo es.
Prueba: Lo primero que vamos a hacer, es lo que se llama como una exploracin
matemtica, comenzaremos con lo que se quiere deducir
|(f g)(x) (f g)(a)| = |f (x)g(x) f (x)g(a) + f (x)g(a) f (a)g(a)|
= |f (x)(g(x) g(a)) + f (a)(f (x) f (a))|
|f (x)||g(x) g(a)| + |g(a)||f (x) f (a)|
< |f (x)||g(x) g(a)| + (|g(a)| + 1)|f (x) f (a)|
Note como se ha sumado y restado al mismo tiempo la expresin f (x)g(a), tambin se ha
obtenido <, gracias a que se le ha sumado 1, a la expresin |g(a)|.
En este punto podemos dar las siguientes observaciones:
Como f es continua en x = a, tenemos garantizada la existencia de 1 > 0 tal que
x X, |x a| < 1 |f (x) f (a)| < 1

20

CAPTULO 1. CONTINUIDAD DE FUNCIONES


con ello podemos calcular
|f (x)| = |f (x) f (a) + f (a)| |f (x) f (a)| + |f (a)| < 1 + |f (a)|
Debido a que g es continua en a, tenemos existe 2 > 0 tal que
x X, |x a| < 2 |g(x) g(a)| <
note lo peculiar de tomar la expresin


2(1 + |f (a)|)


.
2(1+|f (a)|)

Finalmente, como f es continua en a, deber existir un 3 > 0 de modo que se


cumpla con
x X, |x a| < 3 |f (x) g(a)| <


2(|g(a)| + 1)

Note que a nadie se le va a ocurrir, de un de repente semejante expresiones con el


 > 0, todo tiene la explicacin que antes se ha trabajado y se ha visto conveniente
tomar o considerar tales expresiones.
Finalmente bastar considerar el siguiente
= mn{1 , 2 , 3 }
Quedar para el estudiante el enlazar este sistema de ecuaciones para poder demostrar
que el producto f g es una funcin continua en x = a.
Debido a este resultado, y como f (x) = x es continua en cualquier punto de su dominio,
luego f (x) f (x) = x x = x2 es continua, pero podemos utilizando induccin matemtica
sobre n N demostrar que x3 , x4 . . . xn es una funcin continua, como lo muestra en el
resultado siguiente.
Ejemplo 1.24. Demostrar: Sea n N, entonces f (x) = xn es una funcin continua, en
cualquier punto de su dominio.
Prueba: Se sugiere utilizar el Principio de Induccin Matemtico.
Ahora,combinando los resultados anteriores,podemos afirmar que x2 + x + 1, x3 + x2 +
x + 1, x4 + x3 + x2 + x + 1, . . . son funciones continuas. Tambin podemos decir que

21

1.2. LGEBRA DE FUNCIONES CONTINUAS

2x + 1, 2x2 + 3x + 5, 4x3 + 2x2 + 7x + 1, y todas las funciones que se parecen a ellas sern
continuas en todo su dominio.
En general tenemos que toda funcin polinomial es continua en cualquier punto de su
dominio.
Ejemplo 1.25. Sea f (x) = a0 + a1 x + a2 x2 + + an xn una funcin polinomial con
coeficientes en los reales. Demuestre que f es una funcin continua en todo su dominio.
Prueba: Se deja al estudiante.

Divisin de Dos Funciones Continuas


Para poder enfrentar este hecho, trabajaremos en dos partes: primero lo demostraremos
para

1
g

y luego aplicando la multiplicacin de funciones continuas, podemos decir que


f
1
( )f =
g
g

es continua.
Teorema 1.6. Sea g : X R continua en x = a. Si g(a) 6= 0, entonces

1
g

es continua en

x = a.
Prueba: Lo primero es superar el problema del dominio, buscado un subconjunto que
no anule a la imagen, es decir, sea
Y = {x X : g(x) 6= 0}
no hay problema considerar X Y como el conjunto X, para s poder trabajar con este
nico dominio luego sobre este nuevo dominio debemos trabajar. Considere
1
1
|g(x) g(a)|
|( )(x) (
)(a)| =
g
g(a)
|g(x)||g(a)|
Por otro lado como g es continua en x = a, podemos tomar  =
existir 1 > 0 de modo que
x X, |x a| < |g(x) g(a)| <

|g(a)|
2

|g(a)|
,
2

luego debe

22

CAPTULO 1. CONTINUIDAD DE FUNCIONES

por lo tanto
|g(a)| |g(x)| |g(x) g(a)| <

|g(a)|
2

es decir
|g(a)|
< |g(x)|
2
entonces

1
|g(x)|

<

2
.
|g(a)|

Por lo tanto, utilizando la primera parte


1
1
|g(x) g(a)|
|( )(x) (
)(a)| =
g
g(a)
|g(x)||g(a)|
|g(x) g(a)| 2
.
<
|g(a)|
|g(a)|
|g(x) g(a)|.2
=
|g(a)|2
Ahora como  > 0 es arbitrario, luego existe 2 > 0 de modo que x X, |x a| < 2
|g(x) g(a)| <

.|g(a)|2
.
4

Considerando finalmente el valor adecuado, sera


= mn{1 , 2 }
Claro est que lo anterior es un bosquejo de la demostracin formal, se deja al estudiante
dicha verificacin, con todo lo anterior es suficiente para tener xito en ella.
Con el teorema anterior y utilizando el teorema de la multiplicacin de dos funciones
continuas, se demuestra para el caso de la divisin de dos funciones continuas, como se
menciona en el siguiente teorema, cuya demostracin lo dejaremos.
Teorema 1.7. Si f, g : X R continuas en x = a, con g(a) 6= 0, entonces

f
g

es continua

en a.
Prueba: Detallar.
Como una extensin al resultado anterior, es que si tenemos la divisin de dos funciones
polinomiales, por todo lo anterior, se tiene que es una funcin continua en cualquier punto
del dominio en comn que no anule el polinomio que acta como denominador.

23

1.2. LGEBRA DE FUNCIONES CONTINUAS

Ejemplo 1.26. Considere f (x) = a0 + a1 x + a2 x2 + an xn y g(x) = b0 + b1 x + b2 x2 +


+ bm xm dos funciones polinomiales y g(a) 6= 0. Demostrar que

f (x)
g(x)

es una funcin

continua en a.
Prueba: Se deja al estudiante.
Ejemplo 1.27. Determinar un ejemplo de dos funciones , que sean discontinuas en el
punto x = a, pero que su producto sea una funcin continua en dicho punto.
Prueba: La estrategia la conocemos, podemos considerar las dos siguientes funciones

1
,x 0
f (x) =

1 , x < 0
tambin la siguiente funcin g

g(x) =

,x = 0

2 , x 6= 0
las cuales son discontinuas en x = 0, pero la funcin f g queda definida por
x R, (f g)(x) = 2
la cual es una funcin constante, la cual es continua en todo los reales, en particular en
x = 0.
Ahora si podemos dar un estilo de demostracin de algunas situaciones que se ha
dejado en prrafos anteriores.
Ejemplo 1.28. Sea f una funcin definida por f (x) =

4 , x = a

. Pruebe que f es

1 , x 6= a
discontinua, es decir no continua, en x = a.
Prueba: Utilizaremos para ello el Mtodo de Contradiccin, la cual consiste en suponer
que f si sea continua en a.
Consideremos  = 1, luego existe > 0 de modo que x R y |x a| < , entonces
|f (x) f (a)| < 1, es decir |f (x) 4| < 1.

24

CAPTULO 1. CONTINUIDAD DE FUNCIONES


Por otro lado tenemos por densidad la existencia de x0 6= a de modo que
a < x0 < a +

esto nos indica que |x0 a| < , y por lo anterior tenemos |f (x0 ) 4| < 1, luego
|1 4| < 1
la cual es una expresin imposible o contradictoria.
Finalmente podemos indicar que f no puede ser continua en x = a.
Dejaremos el siguiente ejercicio para su consideracin.

2 , x = 4
Ejemplo 1.29. Considere la funcin g(x) =
. Pruebe que g es discontinua

3 , x 6= 4
en x = 4.
Prueba: Detallar.
Un ejercicio con cierta variante, pero que a travs de l, podamos experimentar otra
estrategia de solucin es la siguiente.

Ejemplo 1.30. Sea f una funcin definida por f (x) =

,x = 0

. Demostrar que f

2x , x 6= 0
es una funcin no continua en x = 0.
Prueba: Verificar.
Que cambios mnimos le haras, para que sea continua en x = 0?

1.3.

Continuidad y las Desigualdades

A continuacin se tiene una serie de propiedades que relacionan las funciones continuas
con la relacin de orden menor o mayor, las cuales sern un grupo de propiedades muy
tiles para ms adelante.

1.3. CONTINUIDAD Y LAS DESIGUALDADES

25

Teorema 1.8. Sea f : X R una funcin continua en x = a. Si f (a) > 0, entonces


existe > 0 tal que
x X, |x a| < f (x) > 0.
Prueba: Por dato f es continua en a, luego podemos considerar estratgicamente  =
f (a) > 0, y aplicar la defincin de continuidad, la cual garantiza la existencia de > 0 de
modo que: x X, |x a| < , implica que
|f (x) f (a)| < f (a) = 
con lo cual obtenemos f (a) < f (x) f (a) < f (a), es decir 0 < f (x).
De manera muy similar se puede demostrar, la propiedad siguiente.
Teorema 1.9. Sea f : X R una funcin continua en x = a. Si f (a) < 0, entonces
existe > 0 tal que
x X, |x a| < f (x) < 0.
Prueba: He de esperar que fuese as, se dejara al lector.
Ahora hagamos en general para algn k y ya no con el cero.
Teorema 1.10. Sea f : X R una funcin continua en x = a. Si f (a) > k, entonces
existe > 0 tal que
x X, |x a| < f (x) > k.
Prueba: Podemos considerar la siguiente funcin g : X R definida por
g(x) = f (x) k
esta es la parte muy importante de la presente demostracin, acerca de la creacin de una
nueva funcin que va a solucionar nuestro problema satisfactoriamente. Por dato tenemos
que f (a) > k, luego f (a) k > 0, es decir g(a) > 0, y como g es una funcin continua en
a, gracias a ello tenemos la existencia de un > 0, de modo que
x X, |x a| < g(x) > 0
es decir f (x) k > 0, por lo tanto f (x) > k.

26

CAPTULO 1. CONTINUIDAD DE FUNCIONES


Creemos que la demostracin anterior nos deja una buena estrategia de demostracin,

utilizando una funcin como empalme para aplicar algn resultado ya demostrado. Dejaremos el siguiente resultado, para una exploracin parecida a lo anterior.
Teorema 1.11. Sea f : X R una funcin continua en a. Si f (a) < k, entonces existe
> 0 tal que
x X, |x a| < f (x) < k.
Prueba: Detallar.
Ahora juntaremos los resultados anteriores para originar uno nuevo y de cobertura
ms amplia.
Teorema 1.12. Sea f : X R una funcin continua en x = a. Si f (a) 6= 0, entonces
existe > 0 tal que
x X, |x a| < f (x) 6= 0.
Prueba: Por hiptesis o dato tenemos que f (a) 6= 0, luego se tiene nicamente dos casos:
Si f (a) < 0, luego por uno de los teoremas anteriores tenemos la existencia de un
> 0 tal que:
x X, |x a| < f (x) < 0
con lo cual f (x) 6= 0
Si f (a) > 0, se procede muy similar.

Teorema 1.13. Sea f : X R una funcin continua en a. Si f (a) 6= k, entonces existe


> 0 tal que
x X, |x a| < f (x) 6= k.
Prueba: Se deja al estudiante.

1.4. CONTINUIDAD Y LA COMPOSICIN DE FUNCIONES

1.4.

27

Continuidad y la Composicin de Funciones

En este punto establezcamos lo que sucede con la composicin de dos funciones continuas en un punto, en la sospecha est que debe ser tambin continua, pero la demostracin
de tal afirmacin nos dar algunas estrategias para manejar la continuidad aplicada a la
composicin de funciones.
Teorema 1.14. Sea f : X R,g : Y R, Y Im(f ), a X. Si f es continua en x = a
y g es continua en f (a), entonces g f es tambin una funcin continua en el punto a.
Prueba: Tomamos un  > 0 totalmente arbitrario, debido a que g es continua en f (a),
existe 1 > 0 tal que
y Y, |y f (a)| < 1 |g(y) g(f (a))| < 

(1.20)

por otro lado, como f es tambin continua pero en x = a, se tiene la existencia de 2 > 0,
de modo que
x X, |x a| < 2 |f (x) f (a)| < 1

(1.21)

podemos finalmente considerar, el 2 > 0, para demostrar la continuidad de la composicin, veamos si este valor es el adecuado.
Sea x X = Dom(g f ) y |x a| < 2 , por la ecuacin (1.21) tenemos |f (x) f (a)| < 1 ,
como f (x) Im(f ) Y y |y0 f (a)| < 1 cumple con la ecuacin (1.20) luego
|g(y0 ) g(f (a))| < 
por lo tanto |g(f (x)) g(f (a))| < , luego |g f (x) g f (a)| < .
Si consideramos la continuidad de una funcin, pero sobre un punto aislado, se tiene el
buen resultado que siempre es continua dicha funcin, para ello recordemos lo que significa
ser punto aislado.
Definicin 1.3. Sea X R, x0 X.
x0 es un punto aislado si y solo si existe > 0 tal que (x0 , x0 + ) X = {x0 }.
Teorema 1.15. Sabiendo que x0 es un punto aislado de X, cualquier funcin f : X R
es continua en x0 .

28

CAPTULO 1. CONTINUIDAD DE FUNCIONES

Prueba: Basta considerar el > 0 que define el aislamiento del punto, para cualquier 
que se tome, y cumplir as la definicin de continuidad.
Como una aplicacin al teorema anterior tenemos el siguiente ejemplo.
Ejemplo 1.31. Sea f : Z R. Probar que f es continua en todo punto del dominio.
Prueba: Considere x0 Z un punto cualquiera, demostremos que f es continua en x0 .
Sea  > 0 arbitrario, tomamos adrede =

1
2

> 0, y sea x Z con

|x x0 | <

1
2

Note que x0 1 < x0 12 < x < x0 + 12 < x0 + 1, por lo tanto x0 1 < x < x0 + 1, ahora
como x Z solo puede tomar un solo valor, para x = x0 .
Por lo tanto calculando |f (x) f (x0 )| = |f (x0 ) f (x0 )| = 0 < .
Note que este resultado es independiente de la regla de correspondencia de la funcin,
la grfica de dicha funcin son como punto separados en el plano cartesiano. Anteriormente
se ha utilizado el concepto de continuidad en un conjunto, definamos a continuacin ello.
Definicin 1.4. Sea f : X R una funcin, A X. Diremos que f es continua en A si
y solo si f es continua en cada punto del conjunto A.
Segn el ejemplo anterior, podemos decir que f es una funcin continua en todo Z.
Es costumbre decir que f es continua, cuando es continua sobre todo su dominio.
Prueba: Sea f : A R una funcin continua y B A no vaco. Entonces la funcin f
restringida a B sigue siendo continua.
Sea x0 B un elemento cualquiera, veamos f /B : B R es continua en x0 .
Sea  > 0 arbitrario, de donde buscaremos el > 0?, como f es continua en x0 A,
existe un > 0 tal que
x A, |x x0 | < |f (x) f (x0 )| < 
Considerando el de lo anterior, y sea x B, |x x0 | < , entonces podemos calcular
|f /B(x) f /B(x0 )| = |f (x) f (x0 )| < 

29

1.4. CONTINUIDAD Y LA COMPOSICIN DE FUNCIONES

Lmites y Continuidad
Cuando el punto en donde la funcin es continua, es un punto de acumulacin del
dominio (a X 0 ), la definicin se transforma al concepto de lmite, veamos este buen
resultado a continuacin.
Teorema 1.16. Sea f : X R continua en x = a. Si a X 0 , entonces
f (a) = lmxa f (x).
Prueba: Sea  > 0 cualquiera, como f es continua en a, existe un > 0 de modo que
x X, |x a| < |f (x) f (a)| < 

(1.22)

considerando este > 0 encontrado, procedemos a verificar la defincin del lmite de una
funcin.
Sea x X, 0 < |x a| < x X, |x a| < , por lo tanto cumple la ecuacin (1.22),
es decir
|f (x) f (a)| < 
por lo tanto tenemos que lmxa f (x) = f (a).
Queda claro que solamente se puede utilizar este resultado, cuando se tenga de por
medio punto de acumulacin, es decir si f : X R es continua en x = a, no podemos
asegurar que lmxa f (x) = f (a), mucho menos, desconocemos acerca de la existencia o
no, del lmite de dicha funcin cuando el punto tiende a a.
Por ejemplo, considere X = (1, 2) {4} y f : X R definida por f (x) = x + 1, entonces
f es continua en x = 4, pero claro esta que no existe lmx4 f (x).
Ahora veamos el resultado mas completo, sobre la continuidad en relacin con el lmite.
Teorema 1.17. Sea f : X R una funcin, x0 un punto de acumulacin del conjunto
X.
Entonces f es continua en x0 si y solo si lmxx0 f (x) = f (x0 ).
Prueba: () Tenemos que demostrar que lmxx0 f (x) = f (x0 ). Note que tiene sentido
hablar de dicho lmite ya que x0 X 0 .

30

CAPTULO 1. CONTINUIDAD DE FUNCIONES

Sea  > 0 arbitrario, por la definicin de continuidad tenemos la existencia de un > 0


de modo que
x X, |x x0 | < |f (x) f (x0 )| < 

(1.23)

Luego podemos considerar x X y 0 < |x x0 | < , este valor de x cumple con la


ecuacin (1.23) por lo tanto
|f (x) f (x0 )| < 
()
Tenemos para demostrar que f es continua en x0 .
Considere  > 0 dado arbitrariamente, ahora como lmxx0 f (x) = f (x0 ) luego existe
> 0 tal que
x X, 0 < |x x0 | < |f (x) f (x0 )| < 

(1.24)

Por lo tanto consideremos x X y |x x0 | <


Si x = x0 , podemos calcular
|f (x) f (x0 )| = |f (x0 ) f (x0 )| = 0 < 
Si x 6= x0 , por lo tanto 0 < |x x0 | < , cumple con la ecuacin (1.24) por lo tanto
|f (x) f (x0 )| < 
Por lo tanto, para cualquier caso se cumple con
|f (x) f (x0 )| < 

Considere el siguiente ejemplo, donde se muestra una estrategia de demostracin a


considerar para futuros ejercicios.

0 , x < 0
. Probar que f es discontinua en x = 0.
Ejemplo 1.32. Sea f (x) =

1 , x 0

1.4. CONTINUIDAD Y LA COMPOSICIN DE FUNCIONES

31

Prueba: Consideremos para este problema el mtodo de contradiccin, es decir supngase


que f sea continua en x = 0, por lo tanto tomemos un  = 12 , luego existe > 0 de modo
que:
Si x Dom(f ) = R, |x 0| < , implica que se cumple con
|f (x) f (0)| <

1
2

Simplificando tenemos, si x R, |x| < |f (x)1| < 12 , considerando un real particular


clave para el problema
x0 =

la cual cumple con la ecuacin anterior, es decir |f (x0 )1| < 21 , como f (x0 ) = f ( 2 ) = 0,
es decir
|0 1| <
es decir 1 <

1
2

1
2

lo cual es imposible o un hecho contradictorio.

Por lo tanto, la dificultad se encuentra en la suposicin hecha, negndolo se tiene que


f no es continua en 0.
Nota: Segn la relacin de continuidad y lmite, nos permite dar otra tcnica de
demostracin para estos tipos de ejercicios, como x0 es un punto de acumulacin del
Dom(f ) = R, la continuidad puede ser vista en trminos de lmites.
Calculemos
lmx0 f (x) = 0.
lmx0+ f (x) = 1
por lo tanto no existe el lmx0 f (x), luego por el teorema f no es continua puntual en
x = 0.
Ejemplo 1.33. Encontrar las condiciones necesarias para que la funcin

f1 (x) , x > a
f (x) =

f2 (x) , x a
sea continua en el punto x = a.

32

CAPTULO 1. CONTINUIDAD DE FUNCIONES

Prueba: Basta con que cumpla con


lmxa+ f (x) = f (a), es decir lmxa+ f1 (x) = f2 (a).
lmxa f (x) = f (a), es decir lmxa f2 (x) = f2 (a).
o lo que otros lo indican por
existe lmxa+ f (x);
existe lmxa f (x);
lmxa+ f (x) = lmxa f (x) = f (a).

Ejemplo 1.34. Sea g : R R definida por g(x) = x2 . Probar utilizando la definicin


que g es continua en todo los nmeros reales.
Prueba: Este problema lo dividiremos es dos partes:
Una primera parte informal y de intento.
La segunda es la formalizacin de lo que se ha intudo en la primera parte.
Sea x0 R un punto arbitrario.
Por demostrar que g es continua en x0 .
Sea  > 0 cualquiera, busquemos el > 0 adecuado para la continuidad
|g(x) g(x0 )| = |x2 x20 | = |x + x0 ||x x0 |
Consideremos un 1 = 1, por lo tanto |x x0 | < 1 es decir
|x| |x0 | < |x x0 | < 1
por lo tanto |x| < |x0 | + 1, ahora |x + x0 | |x| + |x0 | < 2|x0 | + 1, luego
|x + x0 ||x x0 | < (2|x0 | + 1)|x x0 | < 
por lo tanto |x x0 | <


2|x0 |+1

= 2 . Hasta aqu la primera parte, se ha logrado obtener los

dos radios 1 y 2 adecuados, vamos a considerar el menor de ellos para nuestro propsito.

33

1.4. CONTINUIDAD Y LA COMPOSICIN DE FUNCIONES


Formalizando:
Sea  > 0 arbitrario, podemos considerar
= min{1,


}
2|x0 | + 1

Veamos si es el adecuado, sea x R, |x x0 | < , note que


|x x0 | < 1
por lo tanto |x x0 | < 1, es decir |x| |x0 | ||x| |x0 || |x x0 | < 1, entonces
|x| < |x0 | + 1, ahora
(1.25)

|x + x0 | |x| + |x0 | < |x0 | + 1 + |x0 | = 2|x0 | + 1


Tambin tenemos |x x0 | <


,
2|x0 |+1

finalmente podemos operar

|g(x) g(x0 )| = |x + x0 ||x x0 | < (2|x0 | + 1)|x x0 | < 

Observacin:
Si queremos aplicar lmites, esto resultar ms sencillo, pero antes note que x0 es un
punto de acumulacin del dominio de la funcin g, por lo tanto tiene sentido intentar con
lmites
lm g(x) = x20 = g(x0 )

xx0

luego g es continua en x0 .
Se debe considerar los dos tipos de resoluciones, ya que en cada uno de ellos presenta
situaciones matemticas que a nosotros nos conviene entenderlas.
Ejemplo 1.35. Considere la siguiente funcin f definida por f (x) =

x + 1 , x > 4

Demostrar que f es una funcin continua en x = 4.

,x 4

34

CAPTULO 1. CONTINUIDAD DE FUNCIONES

Prueba: Debido a que 4 es un punto de acumulacin del dominio de f , utilizaremos


lmites, en particular los lmites laterales:
lm f (x) = lm+ (x + 1) = 5

x4+

x4

tambin
lm f (x) = lm (5) = 5

x4

x4

luego tenemos lmx4 f (x) = 5 = f (4), por lo tanto f es continua en 4.

1.5.

Continuidad y las Sucesiones

El siguiente resultado dentro de la teora de las funciones continuas, es suma importancia, ya que va a conectar dos estructuras matemticas, una estudiada (sucesiones) y la
otra en proceso de desarrollo (continuidad), muchos de los ejercicios que se va a desarrollar
se utilizara este buen resultado. Se trata de una bicondicional, y como sabemos es obligatorio demostrarlo dos doble implicacin: necesidad y suficiencia, uno de tales es un poco
menos dificultoso, pero el otro, se realizara por suposicin contraria, veamos entonces este
teorema.
Teorema 1.18. Sea f : X R una funcin y x0 X. Entonces f es continua en x0 si y
solo si
lm f (xn ) = f (x0 )

,siempre que (xn ) es una sucesin en X con lm xn = x0 .


Prueba: Lo primero es entender que datos tenemos y que es lo que me piden, describiremos ello por medio de las expresiones
Hiptesis.
Tesis.
()
Hiptesis:
f : X R.

1.5. CONTINUIDAD Y LAS SUCESIONES

35

x0 X.
f es continua en x0 .
Considere una sucesin arbitraria (xn ) en X de modo que lm xn = x0 .
Tesis: lm f (xn ) = f (x0 ).
Sea  > 0 tomado en forma arbitraria, ahora debemos buscar un natural n0 que haga
cumplir con la tesis, este es un buen reto, dentro del problema, es ms se podra decir que
es la clave de toda la demostracin. Veamos como buscarlo, debido a que f es continua
en x0 , entonces existe > 0 de modo que
x X, |x x0 | < |f (x) f (x0 )| < 

(1.26)

por otro lado como lm xn = x0 , tiene que existir un n0 N tal que


n > n0 , |xn x0 | <

(1.27)

luego el natural encontrado n0 es, todo parece ser lo que buscbamos.


Ahora debemos comprobar que esto es as, sea m > n0 cualquiera, luego cumple la ecuacin
(1.27)
|xm x0 | <
pero este xm X tambin cumple con (1.26), por lo tanto
|f (xm ) f (x0 )| < 
y esto es justamente el concepto de que lm f (xm ) = f (x0 ).
()
Hiptesis:
f : X R una funcin.
x0 X.
lmn f (xn ) = f (x0 ),siempre que (xn ) es una sucesin en X con lm xn = x0 . Este
dato no significa que exista una sucesin (xn ) que cumpla con ello, no, significa que

36

CAPTULO 1. CONTINUIDAD DE FUNCIONES


en el proceso de la demostracin hay que construir una sucesin (xn ) en X, de modo
que lm xn = x0 , y rein all, se puede utilizar que tal secin cumple tambin con
lm f (xn ) = f (x0 )

Se comprende lo que estamos tratando de decir?, no siga la demostracin si no es


as, detngase primero y piense en ello.
Tesis:f es continua en x0 .
Se sugiere en esta parte utilizar el mtodo de contradiccin , es decir suponer que f
no es continua en x0 , luego hay que negar la defincin de continuidad,sera as
 > 0 / existe x X, |x x0 | < |f (x) f (x0 )| > 
luego considere =

1
n

vamos a poder construir una sucesin (xn ) en X con la caracterstica

que
|x x0 | <

1
n

es decir lm xn = x0 , con la cual ya tenemos para aplicar la hiptesis para esta sucesin y
luego deber cumplir con
lm f (xn ) = f (x0 );
|f (x) f (x0 )| > .
analizando las dos expresiones anteriores, tenemos la contradiccin buscada, meditar en
ello .
Finalmente negando lo que hemos supuesto inicialmente tenemos que f es continua en
x0 .
Sugerencia: Por la importancia de los elementos matemticos que intervienen en esta
demostracin es casi necesario revisar la demostracin anterior con mucha diligencia.

1 , x Q
Ejemplo 1.36. Sea f : R R una funcin definida por f (x) =
.

0 , x I
Pruebe que f no es continua en ningn punto.

37

1.5. CONTINUIDAD Y LAS SUCESIONES

Prueba: Supongamos que exista un nmero real a R, tal que f sea continua en a.
Entonces existe > 0 de modo que
x R, |x a| < |f (x) f (a)| <  =

1
2

ahora tomemos q Q tal que |q a| < , por lo tanto |f (q) f (a)| < 21 , luego
|1 f (a)| <

1
2

(1.28)

tambin podemos considerar r I tal que |r a| < , luego |f (r) f (a)| < 21 , es decir
|f (a)| <

1
2

(1.29)

Por lo tanto podemos hacer el siguiente clculo


1 = |1 f (a) + f (a)| |1 f (a)| + |f (a)| <

1 1
+ =1
2 2

con lo cual, nos resulta una inecuacin imposible o es una contradiccin.


Suficiente, para haber demostrado que f es discontinua en cualquier nmero real.
Observacin: Otra forma de demostrar lo anterior es tambin suponer que exista
a R tal que f es continua en a. En este punto tendemos por otro camino para obtener
el mismo resultado, pero utilizaremos la existencia de una sucesin (xn ) en Q de modo
que lm xn = a, por la continuidad tenemos
lm f (xn ) = f (a)
es decir f (a) = 1.
De otro lado tambin tenemos la existencia, gracias a la densidad de I en R, de una
sucesin (yn ) de modo que lm yn = a, ahora por continuidad tenemos
lm f (yn ) = f (a)
es decir f (a) = 0, este ltimo resultado difiere del ya dado anteriormente (f (a) = 1), por
lo tanto es discontinua.
Ejemplo 1.37. Sea f : R R una funcin definida por f (x) =
Pruebe que f no es continua en x0 = 0.

, x 6= 0

,x = 0

38

CAPTULO 1. CONTINUIDAD DE FUNCIONES

Prueba: Nuevamente utilizaremos este proceso de demostracin que cada vez se esta
volviendo ms eficaz, considere que f sea una funcin continua en x0 = 0. Si considera
 = 1, deber existir un > 0 de modo que
|x 0| < |f (x) f (0)| < 1
es decir
1
|x| < | | < 1
x
considere un real muy particular x1 = mn{ 2 , 21 } esto significa dos cosas
x1 2 ;
x1 12 .
este valor x1 cumple con la ecuacin dada anteriormente, por lo tanto
|

1
1
|<1
<1
x1
x1

por lo tanto 1 < x1 , lo cual no va en lo adecuado con x1 12 .


El estudiante notar la importancia de tomar el punto x1 con la cual se ha obtenido
la contradiccin deseada.
Ejemplo 1.38. Sean f, g : R R dos funciones continuas, tales que x Q, f (x) = g(x).
Demostrar que f = g, es decir x R, f (x) = g(x).
Prueba: Considere x0 I un elemento arbitrario, como x0 Dom0 (f ) = R. Sabemos
que existe una sucesin (xn ) en los nmeros racionales, de modo que
lm xn = x0
ahora, como f y g son continuas en x0 , entonces por propiedad se cumple
lm f (xn ) = f (x0 )

(1.30)

lm g(xn ) = f (x0 )

(1.31)

finalmente podemos indicar que


f (x0 ) = lm f (xn ) = lm g(xn ) = g(x0 )

39

1.5. CONTINUIDAD Y LAS SUCESIONES


por lo tanto se tiene
x I, f (x) = g(x)
y esto lo unimos a nuestra hiptesis obtenemos la igualdad de estas dos funciones
f = g.

Ejemplo 1.39. Sea f : R R una funcin tal que x, y R, f (x + y) = f (x) + f (y).


Suponga que f es continua en 0. Probar que f es continua en toda la recta real.
Prueba: Este ejercicio es importante, ya que la continuidad de un punto asegura la
continuidad en todo R, pero esto solo es posible ya que la funcin preserva sumas, es decir
x, y R, f (x + y) = f (x) + f (y)
claro esta, que no siempre las funciones tiene esta particularidad.
Considere x0 R un real arbitrario, como x0 es un punto de acumulacin del dominio de
f , luego podemos trabajar la continuidad con lmites. Para ello calculemos
lm f (x) = lm f (x0 + h)

xx0

h0

= lm (f (x0 ) + f (h))
h0

= f (x0 ) + lm f (h)
h0

= f (x0 ) + f (0)
= f (x0 + 0)
= f (x0 )
es decir lm x x0 f (x) = f (x0 ), es decir f es continua en x0 .
Este ejercicio es importante, ya que nos revela una tcnica para pasar el lmite de
x x0 a h 0, de este modo nos permite vencer el problema propuesto.
Ejemplo 1.40. Dada la funcin f definida por

sen x , x 6= 0
x
f (x) =

1
,x = 0
La funcin f es continua en todo R?

40

CAPTULO 1. CONTINUIDAD DE FUNCIONES

Prueba: Si uno intuye que no es cierto, debemos encontrar un punto en donde no sea
continua, el punto adecuado es el x = 0, pero caso contrario, debemos demostrar que
efectivamente f es una funcin continua en toda la recta. Optamos por esta ltima afirmacin.
Sea x0 R un real arbitrario, por lo tanto se tiene nicamente dos casos:
Caso 1: Si x0 6= 0, luego podemos calcular
sen x
xx0
x
sen x0
=
x0
= f (x0 )

lm f (x) =

xx0

lm

por lo tanto, en este caso, f es continua en x0 .


Caso 2:Si x0 = 0, podemos calcular
sen x
= 1 = f (0)
x0
x

lm f (x) = lm

x0

estoltimo nos indica que f es continua en x = 0.


Finalmente, como en cualquiera de los dos casos se tiene la misma conclusin, se tiene
que f es continua en toda la recta real.
El estudiante notar, que esta demostracin utiliza la metodologa de caso para su
demostracin, con la caracterstica que se obtiene el mismo resultado al resolver en cada
uno de los casos divididos.
Si nuestra funcin estuviera definida de la forma siguiente

sen x , x 6= 0
x
f (x) =

2
,x = 0
utilizando en ejemplo anterior, deducimos que f es discontinua solamente en el punto
x = 0.
Un ejemplo parecido es el siguiente, claro esta que presentar cierta variante.

41

1.5. CONTINUIDAD Y LAS SUCESIONES

Ejemplo 1.41. Muestre una funcin f : R R que sea discontinua solamente en los
puntos { n1 : n N}.
Prueba: Aqu el problema es directa, me estn pidiendo una funcin que cumpla con
cierta caractarstica, ahora consiste en buscar dicha funcin. Podramos considerar

x , x = 1 , n N
n
f (x) =

0 , x 6= 1 , n N
n
ahora el problema va a consistir en demostrar la condiciones pedidas para dicha funcin.
Sea x0 R cualquiera, luego podemos dividir la demostracin es varios casos:
Caso 1:Si x0 < 0, veamos que f es continua.
Sea  > 0 arbitrario, consideremos = |x0 | = x0 > 0, luego sean x R tal que
|x x0 | < = x0 , entonces x0 < x x0 < x0 , luego
x<0
por lo tanto |f (x) f (x0 )| = |0 0| = 0 < 
Caso 2: Si x0 = 0, veamos que f es continua en x0 . Sea  > 0 arbitrario, por la
propiedad arquimediana tenemos la existencia de un natural n0 R de modo que
0<
tomando =

1
n0

1
<
x0

> 0.

Sea x R, |x x0 | < es decir |x| <

Subcaso 1: Si x =

1
,
m

1
,
n0

ahora en el caso que:

para algn m N, luego


1
1
<
m
n0

luego podemos calcular


|f (x) f (x0 )| = |f (

1
1
1
1
) f (0)| =
0=
<
<
m
m
m
n0

42

CAPTULO 1. CONTINUIDAD DE FUNCIONES


Subcaso 2: Si x 6=

1
, N,luego
m

podemos calcular

|f (x) f (x0 )| = |0 f (0)| = |0 0| = 0 < 


Caso 3: Si x0 <

1
, 1
n0 +1 n0

>, para algn n0 N, se deja al lector para demostrar

que f sigue siendo continua.


Caso 4: Si 1 < x0 , detallar la continuidad en este punto de f , notar que la funcin
en una vecindad de x0 la funcin se anula.
Caso 5:Si x0 = n1 , para algn n N, f es discontinua, puede considerar la tcnica
del lmite, notando que
lm f (x) = 0

xx0

pero f (x0 ) = n1 .

Ejemplo 1.42. Sea f (x) =

sen( 1 ) , x 6= 0
x

y g(x) =

x sen( 1 ) , x 6= 0
x

. Determine

,x = 0
,x = 0
cual o cuales de las funciones dadas, son continuas en 0. Justifique su respuesta.
Prueba: Veamos, con respecto a la funcin f :
Aplicando la equivalencia de continuidad con lmites, ya que 0 es un punto de acumulacin
del dominio, recordemos
f es continua en 0 lm f (x) = f (0)
x0

El problemas es que el siguiente lmite lmx0 f (x) no existe, basta para ello tomar dos
sucesiones que convergen al cero y las dos sucesiones de las imgenes no son convergentes
al mismo punto, o algunas de ellas es divergente. Consideremos
xn =

1
1
, yn =
, n N
2n
(4n + 1) 2

es claro que
lm xn = 0, lm yn = 0
pero
lm f (xn ) = 0 y lm f (yn ) = 1

1.6. CONTINUIDAD Y EL VALOR ABSOLUTO

43

por lo tanto f no es continua en x = 0.


Con respecto a lafuncin g:
Tomando el lmite lmx0 g(x) = lmx0 x sen( x1 ) = 0, es decir
lm g(x) = g(0)

x0

por equivalencia de lmite y continuidad, tenemos que la funcin g es continua en x = 0.


Otra forma de atacar el problema de la continuidad de g es por definicin, para ello
considere  > 0 arbitrario, podemos considerar =  > 0 (es claro que ya se ha intenta
previamente), por lo tanto consideremos x Dom(g) = R, |x 0| < , es decir |x| < ,
finalmente podemos calcular
1
1
|g(x) g(0)| = |x sen( ) 0| = |x|| sen( )| |x| < 
x
x

1.6.

Continuidad y el Valor Absoluto

En esta seccin, nos interesa analizar, si |f | es continua, sabiendo que f es continua, uno
puede intuir que esto es siempre vlido, pero como ocurre muchas veces en las matemticas,
entre lo que parece ser cierto, muchas veces no lo es, o es muy difcil probarlo. Pero en
este caso, si se tiene por cierto, en aquellos puntos en donde la funcin f es continua en
a.
Teorema 1.19. Sea f : X R una funcin continua en a X, entonces |f | es continua
en a.
Prueba: Considere  > 0 arbitrariamente tomado, luego existe > 0 de modo que se
cumpla con:
x X, |x a| < |f (x) f (a)| < 

(1.32)

tomaremos el > 0 encontrado en (1.32) luego, si x X con |x a| < , por lo tanto


podemos calcular
||f (x)| |f (a)|| |f (x) f (a)| < 

44

CAPTULO 1. CONTINUIDAD DE FUNCIONES

Note que en la ecuacin anterior, se ha necesitado el siguiente resultado del valor absoluto
||a| |b|| |a b|

Observacin
1. Un punto clave de la demostracin anterior, ha sido considerar el mismo > 0,
encontrado de la continuidad de f en a.
2. El recproco es verdadero?, todo parece ser que no es cierto, bastar dar un contraejemplo para verificar ello, es decir tenemos que buscar una funcin f , cuyo valor
absoluto |f | es continua, pero f no lo es.
Consideremos
f (x) =

1 , x < 0

,x 0

equivalentemente tenemos
|f (x)| = 1, x R
as tenemos que |f | es una funcin continua en x = 0, pero f no es continua en 0.

x2 9 , x 6= 3
x3
Ejemplo 1.43. Sea f (x) =
. Demostrar que f es discontinua solamente

3
,x = 3
en x = 3.
Prueba: Detallar la demostracin.

Toda funcin Lineal es Continua


Teorema 1.20. Sea f (x) = mx + b, m, b R una funcin, entonces f es continua en
todo punto de R.
Prueba: Sea x0 R un real arbitrario y  > 0 cualquiera, luego consideremos
=


|m| + 2

1.6. CONTINUIDAD Y EL VALOR ABSOLUTO

45

ahora veamos que este valor del tomado, cumple con la definicin de continuidad.
Sea x Dom(f ) = R, |x x0 | < , entonces
|x x0 | <


|m| + 2

Por otro lado calculamos


|f (x) f (x0 )| = |mx + b (mx0 + b)|
= |m||x x0 |
< (|m| + 2)|x x0 |

< (|m| + 2)
|m| + 2
= 

De este modo tenemos que f (x) = 2x + 3, g(x) = 3x + 4 son algunas funciones


continuas en R.

Un Caso Especial
Teorema 1.21. Sean f, g : X R dos funciones continuas en a. Defina : X R como
(x) = max{f (x), g(x)}
, entonces es continua en a.
Prueba: Lo primero es meditar acerca de la funcin , si f (4) = 3 y g(4) = 5, luego
podemos decir que (4) = 5.
Hay algunas cosas que toman ms tiempo de lo que esperamos, si f (x) = 2x + 3 y
g(x) = x2 , Cmo estara definida (x)?
Una forma de atacar el problema es por definicin de continuidad, es decir dado  > 0
obtener > 0 adecuadamente, esto conlleva una dificultad muy interesante, pero busquemos otra, transformemos la funcin en trminos de f (x) y g(x) pero sin utilizar la
expresin mximo.
Afirmamos: (x) = 12 (f (x) + g(x) + |f (x) g(x)|).
En efecto, sea x X dado arbitrariamente, en este punto dividiremos en dos casos:

46

CAPTULO 1. CONTINUIDAD DE FUNCIONES


Caso 1: si f (x) g(x);
Caso 2: si f (x) < g(x).

Caso 1: en este caso se tiene (x) = f (x), tambin tenemos vlido


1
1
(f (x) + g(x) + |f (x) g(x)|) =
(f (x) + g(x) + f (x) g(x))
2
2
= f (x)
Caso 2: si f (x) < g(x) luego tenemos (x) = g(x), por lo tanto
1
1
(f (x) + g(x) + |f (x) g(x)|) =
(f (x) + g(x) + g(x) f (x))
2
2
= g(x)
de los dos casos anteriores tenemos que
1
(x) = (f (x) + g(x) + |f (x) g(x)|)
2
Por otro lado, debido a que f y g son continuas en a, luego f (x) + g(x) es continua,
pero tambn f (x) g(x), y por el teorema de continuidad y valor absoluto, tenemos que
|f (x) g(x)| es continua en a.
Finalmente podemos indicar que
1
(f (x) + g(x) + |f (x) g(x)|)
2
es continua en a, esto quiere decir que es continua en a.
Comentario: Con esta estrategia de demostracin, se ha evitado mucha dificultad,
con respecto a que se hubiera tratado de demostrar por definicon.
Note tambin lo eficaz que ha sino, dividir en dos casos. Un resultado muy parecido pero
utilizando mnimo, lo da el siguiente teorema.
Teorema 1.22. Sean f, g : X R dos funciones continuas en a. Defina : X R como
(x) = min{f (x), g(x)}
, entonces es continua en a.

47

1.6. CONTINUIDAD Y EL VALOR ABSOLUTO


Prueba: Basta utilizar la tcnica del ejercicio anterior, teniendo por cierto que
1
(x) = (f (x) + g(x) |f (x) g(x)|)
2
realice todos los procesos anteriormente dados para verificar con lo pedido.

El siguiente teorema nos da una versin de la composicin de funciones continuas, pero


trabajadas con x +, pero antes recordemos que: Si lmxx0 f (x) = y0 Dom(g) y g
es una funcin continua en y0 , se tiene por vlido que
lm g(f (x)) = g( lm f (x)) = g(y0 )

xx0

xx0

cambiemos un poco en el siguiente resultado.


Teorema 1.23. Sea lmx+ f (x) = y0 Dom(g) y g es continua en y0 . Entonces
lm g(f (x)) = g( lm f (x)) = g(y0 )

x+

Prueba: Sea  > 0 arbitrario, como g es continua en y0 , existe > 0 de modo que:
y Dom(g), |y y0 | < |g(y) g(y0 )| < 

(1.33)

por otro lado tenemos la hiptesis lmx+ f (x) = y0 , aprovechemos ello, se tiene por
vlido la existencia de B > 0 de manera que:
x Dom(f ), x > B |f (x) y0 | <

(1.34)

todo esta listo, intumos que el valor buscado es el B > 0 encontrado anteriormente.
Confirmemos ello, sea x Dom(g f ), x > B, como Dom(g f ) Dom(f ), luego
tendramos
x Dom(f ), x > B
por lo tanto cumple la ecuacin (1.34) entonces |f (x) y0 | < , por otro lado se tiene
que y = f (x) Dom(g) ya que x Dom(g f ), entonces para y = f (x) se cumple con
(1.33), entonces tenemos
|g(f (x)) g(y0 )| < 
esto ltimo quiere decir que |g f (x) g(y0 )| < , y listo.

48

CAPTULO 1. CONTINUIDAD DE FUNCIONES


Observaciones:

* Una de las piezas importante para esta demostracin ha sido el dominio de la composicin g f , recordemos
Dom(g f ) = {x Dom(f ) : f (x) Dom(g)}
* Este ejercicio es muy favorable para nuestro desarrollo abstracto, ya que combina los
temas: continuidad y lmites al infinito, ello nos permite establecer lazos de aprendizaje, necesarios para nuestro desarrollo.
* Qu sucedera si en el teorema anterior, en vez de x + se coloca x , seguir
siendo vlido?
Ejemplo 1.44. Supngase que
f es continua en x0 .
f (x) 0 en alguna vecindad de x0 .
Pruebe que

f es una funcin continua en x0 .

Prueba: Dividiremos el problema en dos partes:


 si f (x0 ) = 0;
 si f (x0 ) > 0.
Caso 1: Si f (x0 ) = 0, entonces tomamos  > 0 arbitrario, ahora debemos buscar el > 0
adecuado, como f es una funcin continua en x0 , entonces podemos encontrar un 1 > 0
tal que:
x Dom(f ), |x x0 | < 1 |f (x) f (x0 )| < 2
debido a que f (x0 ) = 0, tenemos
si x Dom(f ), |x x0 | < 1 |f (x)| < 2

(1.35)

por hiptesis existe un 2 > 0 tal que


x Dom(f ), |x x0 | < 2 f (x) 0

(1.36)

49

1.6. CONTINUIDAD Y EL VALOR ABSOLUTO


pero, si juntamos las ecuaciones (1.35) y (1.36) tenemos un
3 = min{1 , 2 }
donde si x Dom(f ), |x x0 | < 3 ,implica que f (x) < 2 , es decir
p
f (x) < 
Todo, finalmente quedo listo para considerar = 3 , veamos si resulta con este radio:

si x Dom( f ) ,|x x0 | < = 3 , como Dom( f ) = Dom(f ), podemos calcular


p
p
p
| f (x) f (x0 )| = f (x) < 
Caso 2: si f (x0 ) 6= 0, es decir para nuestro caso f (x0 ) > 0.
Sea  > 0 arbitrario por la continuidad de f en x0 , tenemos que existe 1 > 0 tal que
p
x Dom(f ), |x x0 | < 1 |f (x) f (x0 )| <  f (x0 )

(1.37)

por otro lado por hiptesis del problema existe 2 > 0 tal que si
x Dom(f ), |x x0 | < 2 f (x) 0

(1.38)

tambin como f (x0 ) > 0, y por la continuidad de f en x0 , se tiene la existencia de un


3 > 0 de modo que
f (x) > 0, para x Dom(f ), |x x0 | < 3

(1.39)

finalmente veamos que este valor encontrado es el correcto, si x Dom( f ), |x x0 | < ,


entonces como
p
Dom( f ) Dom(f )
y gracias a las ecuaciones (1.37),(1.38) y (1.39) tenemos
p
p
|f (x) f (x0 )|
p
| f (x) f (x0 )| = p
f (x) f (x0 )
|f (x) f (x0 )|
p
<
f (x0 )
p
 f (x0 )
< p
f (x0 )
= 
note que

f (x) +

p
f (x0 ) > 0 entonces es posible dividir entre este valor.

50

CAPTULO 1. CONTINUIDAD DE FUNCIONES

Observamos: Si queremos simplificar el ejercicio anterior, podemos utilizar la

composicin de funciones, considerando g(x) = x, se tiene por hiptesis que f es


continua en x0 y como f (x0 ) 0, g es continua en f (x0 ), esto quiere decir que g f es

continua en x0 , es decir f es continua en x0 y listo.


Cul de los dos estilos de demostracin nos conviene?, creemos que la respuesta es muy
fcil,los dos, la rapidez de resolver un problema no esta en los objetivos trazados o como
lineamientos de la curricula, en forma contraria muchas veces deteriora las propuestas
iniciales del mismo.
Ejemplo 1.45. Dada la funcin f (x) =

x3 + 2x2 + x. Responda

1. f es continua en x = 1.
2. f es continua en 0.
3. lmx1 f (x) = f (1)?
4. calcule lmx0 f (x).
Prueba: Lo primero que vamos a hacer, es determinar el dominio de la funcin f , para
ello considere
x3 + 2x2 + x 0
x(x2 + 2x + 1) 0
x(x + 1)2 0
entonces x = 1 o x 0, entonces
Dom(f ) = {1} [0; +)
1. Segn lo anterior 1, es un punto aislado del dominio de f , por lo tanto f es continua
en 1.
2. Una manera de ver esto, es por composicin de funciones, considerando
g(x) =

h(x) = x3 + 2x2 + x

1.7. CONTINUIDAD Y LAS FUNCIONES TRIGONOMTRICAS

51

por lo tanto f (x) = g h(x), x Dom(f ), ahora tenemos por vlido que g es
continua en h(0) = 0 y h es continua en x = 0, por lo tanto f es continua en 0.
3. Al ser x = 1 un punto aislado, no tiene sentido expresar lmx1 f (x).
4. ver.

En el ejercicio anterior fue clave analizar el dominio de la funcin dada.

1.7.

Continuidad y las Funciones Trigonomtricas

Consideremos ya definidos las funciones trigonomtricas seno y coseno. Tambin se


tiene que
x R, | sen x| |x| y | cos x| 1
De manera similar tenemos establecido la siguiente identidad
x, y R, sen x sen y = 2 sen(

xy
x+y
) cos(
)
2
2

Veamos que la funcin seno es continua en todo su dominio.


Sea y R arbitrario o cualquiera y sea  > 0, podemos considerar =  > 0.
Sea x R, |x y| < |x y| < , por otro lado, calculemos
|f (x) f (y)| | sen x sen y|
xy
x+y
= |2 sen(
) cos(
)|
2
2
xy
x+y
2| sen(
)|| cos(
)|
2
2
xy
2|
|
2
= |x y|
< 
esto quiere decir que la funcin seno es una funcin continua en todo los reales.
Similarmente, teniendo definido la funcin seno y coseno es posible probar que

52

CAPTULO 1. CONTINUIDAD DE FUNCIONES


x R, | sen x| 1
x, y R, cos x cos y = 2 sen( x+y
) sen( xy
)
2
2

razonando como en el prrafo anterior, tenemos


| cos x cos y| 2 1 |

xy
| = |x y|
2

Por lo tanto podemos indicar que la funcin coseno es continua en todo su dominio.
Teniendo definidos las funciones seno y coseno como dos funciones continuas, podemos
definir nuevas funciones
tan x =

sen x
;
cos x

cot x =

cos x
;
sen x

sec x =

1
;
cos x

csc x =

1
.
sen x

Utilizando el lgebra de funciones continuas, en los puntos en donde estos existan, se


prueba que dichas funciones son continuas, por ejemplo la funcin tangente es continua
en los puntos en donde el coseno no sea cero, es decir el los puntos reales que no tengan
la forma siguiente
(2n + 1)
con n Z.

2
Continuidad Uniforme
Definicin 2.1. Sea f : X R una funcin. Diremos que f es uniformemente continua si
y solo si para cada  > 0 existe un > 0 tal que: x, y X, |xy| < |f (x)f (y)| < .
Una forma de entender el concepto, es dando ejemplos concretos que cumplan y no
verifiquen con la definicin. Pero tambin es negndola, veamos ello.
Sea f : X R una funcin. Diremos que f no es uniformemente continua si y solo si
existe un  > 0 de modo que para todo > 0 es posible encontrar x, y X tal que
|x y| < |f (x) f (y)| 
Note tambin que en la definicin de continuidad uniforme, el gran parecido con la
continuidad en un punto, la diferencia es que no se fija en ningn punto x0 .
Un primer resultado es afirmar que toda funcin uniformemente continua, es continua
en todo su dominio. Esto significa que si tenemos una funcin que no sea continua, es
imposible que sea uniformemente continua, pero si tenemos una funcin continua, es
posible que se den dos casos que sea uniformemente continua o que no lo sea, ejemplo
f (x) = x es continua y uniformemente continua, pero f (x) = x2 es continua y no es
uniformemente continua.
Teorema 2.1. Sea f : X R. Si f es uniformemente continua, entonces f es continua.
Prueba: Sea a X un elemento cualquiera del dominio de f , tenemos que demostrar, f
es continua en a.
Sea  > 0 arbitrario, luego por hiptesis existe un > 0 tal que
x, y X, |x y| < |f (x) f (y)| < 
53

54

CAPTULO 2. CONTINUIDAD UNIFORME

en particular para y = a, tenemos que


x X, |x a| < |f (x) f (a)| < 
esto ltimo me indica que f es continua en a.
Con este buen resultado tenemos que, si queremos buscar ejemplos de funciones uniformemente continuas, tenemos que buscarlas en el conjunto de las funciones continuas.
Con esto tenemos que si una funcin no es continua en un punto x0 , luego tampoco es
uniformemente continua,
en el dominio que contenga a x0 .

1 , x > 1
no es una funcin uniformemente continua, ya que
De este modo f (x) =

2 , x 1
f no es continua en 1.
Expongamos una funcin constante es uniformemente continua.
Ejemplo 2.1. Demuestre que f : R R donde f (x) = k con k constante, es uniformemente continua.
Prueba: Sea  > 0 cualquiera, podemos tomar para este caso = , veamos si cumple
con la deficin. Si x, y R, con |x y| < , entonces calculemos
|f (x) f (y)| = |k k| = 0 < .

Note que en la demostracin anterior se pudo haber considera = 1 o = 2, y


an as funciona el proceso. De lo anterior tenemos que cualquier funcin constante es
uniformemente continua. As tenemos f (x) = 1, f (x) = 2. Ya tenemos un conjunto de
funciones uniformemente continuas.
Tambin tenemos que la funcin identidad lo es.
Ejemplo 2.2. Sea f : R R definida por f (x) = x, es una funcin uniformemente
continua.
Prueba: Considere  > 0 arbitrario, tomemos = , veamos que s es el valor adecuado
que verifique la definin.

55
Sea x, y R de modo que |x y| < , luego
|x y| < 
con lo cual calculando tenemos, |f (x) f (y)| = |x y| < .(listo)
Pero tambin tenemos que:
Ejemplo 2.3. Demuestre que f : R R donde f (x) = 2x, es uniformemente continua.
Prueba: Se deja para su desarrollo, claro est que se debe tomar de referencia el ejemplo
anterior.
Se podr generalizar el resultado anterior, es decir si tenemos la funcin f : R R
definida por
f (x) = x
con R?
Ejemplo 2.4. Demuestre que f : R R donde f (x) = x es uniformemente continua,
para cada R.
Prueba: Sea R arbitrario, luego tenemos dos casos:
Caso 1: si = 0.
En este caso se tiene la funcin nula, la cual, como se demostr es uniformemente continua.
Caso 2: si 6= 0.
Sea  > 0 cualquiera, luego considerando =
Luego calculando


> 0. Sea x, y R con |x y| < =
||

|f (x) f (y)| = |x y| = |||x y| < ||


.
||


= .
||

Con ello, tenemos otro grupo de funciones que son uniformemente continuas, tales

como f (x) = 3x, g(x) = 4x, h(x) = 5x. Veamos otro ejemplo.
Ejemplo 2.5. Pruebe que toda funcin f : Z R, es uniformemente continua.

56

CAPTULO 2. CONTINUIDAD UNIFORME

Prueba: Recuerde que si tenemos dos nmeros enteros x e y con |x y| < 21 , se cumple
que x = y, note que se tiene por vlido que f ya es continua, sin necesidad de saber su
regla de correspondencia.
Ya sabemos que f (x) = x es uniformemente continua, el producto con ella misma, es
decir f (x) = x2 , como se mostrar en el siguiente ejemplo, no es una funcin uniformemente continua.
Ejemplo 2.6. Sea f : (0, +) R definida por f (x) = x2 . Pruebe que f no es uniformemente continua.
Prueba: La estrategia que utilizaremos es el mtodo de contradiccin, supngase que f
es uniformemente continua, luego podemos tomar  = 1, por lo tanto existe > 0 de
modo que
x, y (0, +), |x y| < |f (x) f (y)| < 1
ahora, existe n N tal que 0 <

(2.1)

< n, implica que


n > 1

(2.2)

podemos considerar los siguientes puntos, estratgicamente tomados

x0 = n + , z0 = n
2
por lo tanto
|x0 z0 | <

<
2

de la ecuacin (2.1) consideremos x = x0 e y = z0 , tenemos |x0 z0 | < por lo tanto


|f (x0 ) f (z0 )| = |x20 z02 | = |x0 + z0 ||x0 z0 | = |2n + || | > (2n) = n > 1
2 2
2
por lo tanto
|f (x0 ) f (z0 )| > 1
la cual es incompatible con la ecuacin (2.1).
La estrategia utilizada anteriormente es muy importante retenerla, nos brinda un
modelo para realizar algunas demostraciones, como se muestra en el siguiente ejemplo.

57
Ejemplo 2.7. Sea f : R R una funcin definida por

1
,x 0
f (x) =

1 , x < 0
Probar que f no es uniformemente continua.
Prueba: Supongamos que f si es uniformemente continua, luego podemos considerar
=

1
2

y existe luego un > 0,por lo tanto


x, y R, |x y| < |f (x) f (y)| <

1
2

(2.3)

consideremos x = 4 , f = 4 , con estos valores, tenemos


|x y| =

<
2

luego por la ecuacin (2.3) tenemos


|f (x) f (y)| = 2 <

1
2

lo cual es imposible.
Considere ahora el siguiente ejemplo, muy ilustrativo, deduciendo que la continuidad
uniforme es dependiente tambin del dominio de la funcin que se este considerando.
Teorema 2.2. Sea f : [a, b] R tal que f (x) = x2 . Pruebe que f es una funcin
uniformemente continua.
Prueba: Considere  > 0 arbitrario, luego a x b, a y b, x, y [a, b], por lo
tanto
|x + y| M = max{|2a|, |2b|}
podemos considerar para este caso

M

Sea x, y [a, b] con |x y| < |x y| < =
.
M
Luego podemos calcular
=

|f (x) f (y)| = |x2 y 2 |


= |x + y||x y|

< M
=
M

58

CAPTULO 2. CONTINUIDAD UNIFORME

Segn lo anterior, si nos preguntan, la funcin f (x) = x2 es uniformemente continua?,


nuestra respuesta sera, depende, del dominio que tenga.
Como aplicacin numrica resuelva el siguiente ejercicio.
Ejemplo 2.8. Considere la siguiente funcin f : [0, 1] R definida por f (x) = x2 .
Pruebe que f es uniformemente continua.
Prueba: Se deja para la verificacin utilizando los elementos del ejercicio anterior.
Tambin considere
Ejemplo 2.9. Sea f : [2, 6] R definida por f (x) = x2 . Pruebe que f es uniformemente
continua.
Prueba: Note que |x + y| M = max{4, 12} = 12.

2.1.

Uniformemente Continua y la Sucesin de Cauchy

Recordemos el significado de lo que es una sucesin de Cauchy: Sea (xn ) una sucesin
en R. Diremos que (xn ) es de Cauchy si y solo si para cada  > 0, existe un nmero
natural n0 N tal que
m, n > n0 , |xm xn | < 
Recordar tambin que se tiene un buen resultado sobre este punto, que dice as:
(xn ) es de Cauchy (xn ) es convergente
Tremendo este resultado.
Teorema 2.3. Sea f : X R uniformemente continua y (xn ) una sucesin en X.
Si (xn ) es de Cauchy, entonces (f (xn )) es tambin de Cauchy.
Prueba: Sea  > 0 dado en forma arbitraria, como f es uniformemente continua, existe
> 0 de modo que
x, y X, |x y| < |f (x) f (y)| < 

2.2. UNIFORMEMENTE CONTINUA Y LOS CONJUNTOS ACOTADOS

59

ahora como (xn ) es de Cauchy, existe n0 N tal que


m, n > n0 , |xm xn | <
de lo anterior, se ha encontrado el natural adecuado para nuestro problema, n0 , veamos
que esto es as.
Sea m, n > n0 arbitrario, luego xm , xn cumple con
|xm xn | <
luego
|f (xm ) f (xn )| < 

El recproco del teorema anterior, no es cierto, ya que podemos considerar f : R R


tal que f (x) = 0, es una funcin uniformemente continua y xn = n, f (xn ) es una sucesin
de Cauchy, por ser una sucesin nula, pero (xn ) no es de Cauchy, ya que no es convergente.
Teorema 2.4. Sea f : X R uniformemente continua y (xn ) una sucesin en X.
Si (xn ) es convergente, entonces (f (xn )) es tambin convergente.
Prueba: Esto es cierto, ya que hablar de sucesin de Cauchy, es equivalente a convergente.

2.2.

Uniformemente continua y los conjuntos acotados

En esta seccin tenemos este buen resultado, de anlisis real, que si tenemos una
funcin f : X R uniformemente continua y X es un conjunto acotado en R, tales como
X = [1, 2] o < 0, 12 >, tenemos por cierto que la imagen directa de X, es decir f (X), es
un conjunto acotado.
En su demostacin hace uso de resultados no realizados, pero vale la pena, tenerlo presente,
lo dejaremos como verdadero.
Ejemplo 2.10. Si f : X R una funcin uniformemente continua y X acotado. Entonces
demostrar que f (X) es un conjunto acotado.

60

CAPTULO 2. CONTINUIDAD UNIFORME


Este resultado nos sirve para detectar algunas funciones que no sean uniformemente

continua, por ejemplo.


Ejemplo 2.11. Considere f : h0, 1i R definida por f (x) = x1 . Demuestre que f no es
uniformemente continua.
Prueba: Supngase que si lo fuere, como su dominio X = h0, 1i es acotado, tendra que
serlo la imagen de ella, es decir
f (X) = h1, +i
y no lo es.
Un buen resultado tambin es:
Ejemplo 2.12. Considere f : (a, b) R definida por f (x) = x2 . Demuestre que f es
uniformemente continua.

2.3.

Adicin de Funciones Uniformemente Continuas

He de esperar que si tenemos dos funciones uniformemente continuas, la suma de estas,


tambin lo sean. Luego podemos decir que la adicin es cerrada.
Teorema 2.5. Sean f, g : X R dos funciones uniformemente continuas. Entonces f +g
tambin lo es.
Prueba: Utilizaremos el mtodo directo para su demostracin, para ello considere  > 0
dado arbitrariamente, luego por hiptesis tenemos
1 > 0/ x, y X, |x y| < 1 |f (x) f (y)| <


2

(2.4)

2 > 0/ x, y X, |x y| < 2 |g(x) g(y)| <


2

(2.5)

tambin

como ocurre en esta parte se debe tomar el valor mnimo de estos dos deltas encontrados
= mn{1 , 2 }

2.3. ADICIN DE FUNCIONES UNIFORMEMENTE CONTINUAS

61

Ahora veamos si resulta con el valor considerado, sean x, y X con |x y| < , entonces
se cumple con las ecuaciones (2.4) y (2.5), por lo tanto
|(f + g)(x) (f + g)(y)| = |f (x) f (y) + g(x) g(y)|
|f (x) f (y)| + |g(x) g(y)|


+
<
2 2
= 

Observacin: lo que podemos extraer de esta demostracin, es el hecho de considerar


el mnimo de dos radios dados, es decir = mn{1 , 2 }, tenemos que retener esta accin
que se repite muchas veces en matemtica.
Como aplicacin del teorema anterior es:
Ejemplo 2.13. Sean f, g : [1, 2] R dos funciones definidas por f (x) = x2 y g(x) = 3x.
Demuestre que f + g es uniformemente continua.
Prueba: Aqu tenemos dos caminos a tomar:
El primero de ellos es por defincin de uniformidad continua, es decir con  y .
El segundo, es utilizar los resultados anteriores, ya que sabemos que f y g ambas
son uniformemente continuas, sobre el conjunto compacto [1, 2], luego la suma de
ellos tambin lo es.
Se deja al estudiante que siga ambos caminos.

Ejemplo 2.14. Considere f (x) = x2 + x, x [0, 2]. Demuestre por definicin que f es
uniformemente continua.

62

CAPTULO 2. CONTINUIDAD UNIFORME

2.4.

Sustracin de Funciones Uniformemente


Continuas

Teorema 2.6. Sean f, g : X R dos funciones uniformemente continuas. Entonces f g


tambin lo es.
Prueba: El mtodo o la tcnica de la demostracin es muy parecido a lo demostrado
anteriormente, luego tenemos una buena oportunidad para ejercitarse en ella.
Por hiptesis tenemos
1 > 0/ x, y X, |x y| < 1 |f (x) f (y)| <


2

(2.6)

2 > 0/ x, y X, |x y| < 2 |g(x) g(y)| <


2

(2.7)

tambin

consideramos
= mn{1 , 2 }
Ahora veamos si resulta con el valor considerado, sean x, y X con |x y| < , entonces
se cumple con las ecuaciones (2.6) y (2.7), por lo tanto
|(f g)(x) (f g)(y)| = |f (x) f (y) g(x) + g(y)|
|f (x) f (y)| + |g(x) g(y)|


<
+
2 2
= 

Como se aprecia no hay mucha diferencia con la demostracin anterior, solamente es


un manejo con valor absoluto.

Ejemplo 2.15. Demuestre utilizando la definicin que f : [1, 2] R definida por f (x) =
x2 4x es uniformemente continua.
Prueba: Se deja al estudiante en su resolucin.

2.5. MULTIPLICACIN ESCALAR Y UNIFORMEMENTE CONTINUA

2.5.

63

Multiplicacin Escalar y Uniformemente


Continua

Ahora le toca a la multiplicacin de una funcin uniformemente continua por un


escalar.
Teorema 2.7. Sean R, f : X R una funcin uniformemente continua, entonces
f tambin lo es.
Prueba: Sobre el escalar, podemos dividirla en dos casos:
Si = 0, luego la funcin f es la funcin nula, con lo cual se tiene por cierto lo
pedido,podra justificar este resultado.
Si 6= 0, luego considere  > 0, dado arbitrariamente, luego por dato existe tal que si
x, y X con |x y| < , implique |f (x) f (y)| <


.
||

Luego podemos considerar el mismo encontrado anteriormente, claro esta que tambin se puede utilizar para este propsito
0 < 1 <
Sea x, y X, |x y| < , por lo anterior
|f (x) f (y)| <


||

es decir
|f (x) f (y)| < 
por lo tanto |(f )(x) (f )(y)| < 
Con el resultado anterior, ya tenemos que cualquier combinacin lineal, de funciones
uniformemente continuas, tambin es uniformemente continua. Ello lo mostramos en el
siguiente resultado.
Teorema 2.8. Sean , R, f, g : X R son funcines uniformemente continuas,
entonces f + g tambin lo es.

64

CAPTULO 2. CONTINUIDAD UNIFORME

Prueba: Detallar por medio de los resultados anteriores, pero se sugiere realizarlos por
definicn, es decir por medio de  y .
Con ello ya tenemos una gama de funciones uniformemente continuas:
f (x) = x2 + x, f (x) = 6x2 5x + 4
con dominio [2, 4].
He de preguntarnos si el producto de dos funciones uniformemente continuas, tambin
lo es, la respuesta es que no siempre lo es,para ello daremos un contraejemplo.
Considere las funciones uniformemente continuas f, g : (0, +) R donde f (x) =
g(x) = x, pero
h(x) = f (x) g(x) = x2
sabemos que no lo es.
Ejemplo 2.16. Demuestre que 2f (x) + 3g(x) es uniformemente continua, siendo f, g :
X R uniformemente continuas.
Prueba: Se sugiere realizarlo por definicin.
Ejemplo 2.17. Considere f : X R, g : Y f (X) R dos funciones uniformemente
continuas. Demuestre que g f tambin lo es.
Prueba: Se sugiere realizarlo por definicin.

3
Teoremas Fundamentales de Continuidad
En esta ltima parte, del presente material, expondremos los teoremas fundamentales
para funciones continuas, las cuales nos van a generar implicaciones muy importantes con
aplicaciones al algebra, a la teora de ecuaciones, etc, generando lo fundamental del estudio
de las funciones continuas, usted puede tambin, retener nicamente los resultados, as
poder aplicarlos directamente a algunos problemas que normalmente se nos propone, en
caso contrario puede detallar el proceso demostrativo del mismo, con lo cual es un camino
tambin adecuado, ya que en el proceso de demostracin, se notara las tcnicas propias
de este tema.
Teorema 3.1. Si f es continua en [a, b] y f (a) < 0 < f (b), entonces existe c (a, b) tal
que f (c) = 0.
Prueba: Considere el siguiente conjunto
C = {x [a, b] : f (x) < 0}
Veamos que C 6= : basta notar que x = a C.
C es un conjunto acotado superiormente: basta considerar
C [a, b]
como R es un cuerpo completo, esta garantizado c = sup C (a c b).
Veamos que f (c) 0: Para ello sea n N, luego existe xn C tal que
c

1
< xn c
n
65

66

CAPTULO 3. TEOREMAS FUNDAMENTALES DE CONTINUIDAD


entonces lm xn = c, ahora como f es continua, luego
lm f (xn ) = f (c)
ahora como xn C, entonces f (xn ) < 0, por lo tanto
lm f (xn ) lm 0
es decir f (c) 0.
Ahora veamos que 0 f (c): como f (c) 0 < f (b), entonces c < b, es decir bc > 0,
por arquimediano existe n0 N de modo que
1
<bc
n0
es decir c +

1
n0

< b.

Sea n > n0 arbitrario, luego

1
n

<

1
,
n0

c+

entonces c +

1
n

<c+

1
,
n0

es decir

1
< b, n > n0
n

consideremos
zn = c +

1
n0 + n

luego lm zn = c, por lo tanto c < zn , es decir


a c < zn b
como f es continua en x = c, luego tenemos
lm zn = c
por lo tanto
lm f (zn ) = f (c)
como c = sup C < zn , entonces zn 6 C, es decir f (zn ) 0, por lo tanto
lm f (zn ) 0
finalmente f (c) 0.
Con todo lo anterior tenemos la existencia de un c (a, b) tal que
f (c) = 0

67

Cuando tenemos f (x) = x3 , la cuales continua sobre [1, 1], y como se tiene
f (1) < 0 < f (1)
por el teorema, esta garantizada la existencia de un elemento c (1, 1) de modo que
f (c) = 0
esto es cierto, basta tomar el c = 0.
Si consideramos la siguiente funcin f : [1, 1] R definida por

4
,x 0
f (x) =

2 , x < 0
es claro que se cumple con
f (1) < 0 < f (1)
pero no existe c (1, 1) de modo que f (c) = 0, el problema es que f no es continua en
[1, 1], falla la continuidad en x = 0.
Un teorema casi equivalente tenemos
Teorema 3.2. Si f es continua en [a, b] y f (b) < 0 < f (a), entonces existe c (a, b) tal
que f (c) = 0.
Prueba: Claro esta que no hace falta repetir la tcnica par este ejercicio, nicamente
hay que cambiar la funcin, considere
g(x) = f (x)
una funcin que tambin esta definida en [a, b], como g es continua en [a, b] y se cumple
f (b) < 0 < f (a), por lo tanto
f (a) < 0 < f (b)
es decir
g(a) < 0 < g(b)

68

CAPTULO 3. TEOREMAS FUNDAMENTALES DE CONTINUIDAD

por lo tanto, por el teorema anterior, existe c (a, b) tal que g(c) = 0, es decir f (c) = 0,
por lo tanto
f (c) = 0

El estudiante notar lo eficaz que es utilizar lo demostrado anteriormente y como se


adeca al nuevo modelo. la tcnica utilizada genera una nueva funcin g(x) = f (x) la
cual, es la clave del teorema anterior.
El siguiente teorema generaliza el resultado anterior, pero ya no con f (a) < 0 < f (b)
sino mediante un elemento k
f (a) < k < f (b)
Teorema 3.3. Si f es continua en [a, b] y f (a) < k < f (b), entonces existe c (a, b) tal
que f (c) = k.
Prueba: Podemos considerar esta parte una funcin g(x) = f (x) k, la cual es continua
en [a, b], y se tiene
g(a) = f (a) k.
g(b) = f (b) k.
como f (a) < k < f (b) se tiene f (a) k < 0 < f (b) k, por lo tanto
g(a) < 0 < g(b)
por lo tanto existe c (a, b) tal que g(c) = 0, es decir f (c) k = 0, por lo tanto
f (c) = k

Note la gran simplicidad de la demostracin, todo gracias a que se ha demostrado


el primer resultado, que ha sido muy laborioso. Algunos a estos teoremas le llaman El
teorema del Valor Intermedio.
Daremos a continuacin algunas aplicaciones a los resultados anteriores.

69

Aplicacin a las ecuaciones


Ejemplo 3.1. Demostrar que la ecuacin x3 + 3x = 2 tiene una solucin real entre 0 y 1.
Prueba: Basta considerar f (x) = x3 + 3x 2, la cual es una funcin continua sobre [0, 1]
y tenemos
f (0) = 2 < 0 < f (1) = 2
por lo tanto existe c (0, 1) de modo que f (c) = 0 es decir
x3 + 3x 2 = 0

Realizando un clculo aproximado tenemos que c 0, 59607.


Note tambin que se pudo hacer utilizado otros valores, por ejemplo f ( 12 ) = 38 y
f (1) = 2, por lo tanto
1
f ( ) < 0 < f (1)
2
aplicando el teorema respectivo, tenemos que existe c ( 21 , 1) tal que f (c) = 0, es claro
que basta tomar el mismo c ya que c (0, 1).
Considere como practicando el siguiente ejemplo.
Ejemplo 3.2. Demostrar que x4 + x3 2x2 + x = 4 tiene una solucin real c, tal que
0 < c < 2.
Prueba: Se deja al estudiante, este procedimiento.
Ejemplo 3.3. Pruebe que la ecuacin x cos x = 0 tiene solucin entre x = 0 y x = 2 .
Prueba: Es natural considerar la siguiente funcin f (x) = x cos x definida en [0, 2 ], la
cual es continua en dicho intervalo y tambin se tiene
f (0) = 0 cos 0 = 1

f ( ) = cos =
2
2
2
2

70

CAPTULO 3. TEOREMAS FUNDAMENTALES DE CONTINUIDAD

por lo tanto tenemos que f (0) < 0 < f ( 2 ), por lo tanto debe existir c (0, 2 ) de modo
que f (c) = 0, es decir
c cos c = 0
por lo tanto existe una raz en los lmites pedidos.
La solucin aproximada es x 0, 739085.

Un problema importante
Teorema 3.4. Sean f y g dos funciones continuas en [a, b] tal que a < b, de modo que se
cumpla con f (a) < g(a) y f (b) > g(b).
Entonces existe c R con a < c < b tal que f (c) = g(c).
Prueba: Considere la siguiente funcin h(x) = f (x) g(x), x [a, b], es claro que h es
continua en [a, b] y
h(a) = f (a) g(a) < 0 < f (b) g(b) = h(b)
por lo tanto del teorema del valor intermedio se tiene la existencia de c (a, b) de modo
que h(c) = 0, es decir f (c) g(c) = 0, por lo tanto
f (c) = g(c)

Note la importancia de construir una funcin, para nuestro caso h(x), que esta muy
relacionada a la informacin dada por el teorema.

Un problema especial
Teorema 3.5. Sea f : [0, 1] (0, 1) una funcin continua. Entonces existe c (0, 1) de
modo que f (c) = c.
Prueba: Nuevamente las condiciones del teorema, nos permite construir una funcin
adecuada
g(x) = f (x) x

71
la cual es continua en [0, 1], y tambin se tiene
g(1) = f (1) 1 < 0 < f (0) = f (0) 0 = g(0)
ahora por el teorema de valor intermedio, se tiene garantizada la existencia de un c (0, 1)
tal que g(c) = 0, es decir
f (c) c = 0
por lo tanto, tenemos lo pedido f (c) = c.
Note nuevamente la importancia de dar la funcin g(x) = f (x) x adecuadamente.
Teorema 3.6. Sea f : [a, b] R una funcin continua, tal que f (a) > a y f (b) < b.
Entonces existe c (a, b) de modo que se cumpla con f (c) = c.
Prueba: Se deja al lector, para su resolucin.

Aplicacin a las ecuaciones polinomiales


Teorema 3.7. Demostrar que todo polinomio con coeficientes en los reales, y de grado
impar, posee al menos una raz real.
Prueba: Considere tal polinomio p(x) = a0 + a1 x + + an xn , an 6= 0.
Caso 1: si an > 0, por lo tanto el polinomio puede expresarse tambin as
p(x) = xn (

a0
a1
an1
+ n1 + +
+ an )
n
x
x
x

debido a que n es impar y an > 0 tomando lmite al infinito, tenemos que


lm p(x) = +

x+

por lo tanto existe A > 0 tal que x > A, entonces


p(x) > 1
Pero tambin se tiene
lm p(x) =

72

CAPTULO 3. TEOREMAS FUNDAMENTALES DE CONTINUIDAD

es decir existe B > 0 tal que x < B, entonces se cumple


p(x) < 1
Tomando x1 = A + 1, entonces se cumple
p(x1 ) = p(A + 1) > 1
tambin tenemos, consideremos x2 = B 1, entonces
p(x2 ) = p(B 1) < 1
podemos considerar la funcin p : [x2 , x1 ] R la cual es una funcin continua tal que
p(x2 ) < 1 < 0 < 1 < p(x1 )
finalmente por el teorema del valor intermedio tenemos la existencia de x0 (x2 , x1 ) tal
que
p(x0 ) = 0
por lo tanto x0 es una raz que cumple con las condiciones pedidas.
Caso 2: si an < 0, dejaremos al estudiante la verificacin del mismo.
Ejemplo 3.4. Sea f : [3, 4] [3, 4] una funcin continua. Pruebe que existe c (3, 4)
tal que f (c) = c.
Prueba: Detallar.
Teorema 3.8. Sea f : [0, 1] R una funcin continua, con f (0) = f (1). Entonces existe
x [0, 1] tal que
1
f (x) = f (x + )
2
Prueba: Ya sabemos la tcnica de este tipo de problemas, es necesario formar o construir
una funcin adecuada, para nuestro caso, observando lo que me piden se tiene
1
h(x) = f (x) f (x + )
2
para todo x [0, 12 ].
Si f (0) = f ( 12 ), luego podemos tomar el valor de x0 = 0, luego
1
f (x0 ) = f (x0 + )
2

73

3.1. CONTINUIDAD Y LOS INTERVALOS


Ahora en el caso en que f (0) 6= f ( 12 ), podemos calcular
1
1
h(0) = f (0) (0 + ) = f (0) f ( )
2
2
1
1
1 1
1
h( ) = f ( ) f ( + ) = f ( ) f (0)
2
2
2 2
2
ntece que h(0) 6= h( 12 ), luego debe existir c (0, 12 ) tal que h(c) = 0, es decir
1
f (c) f (c + )
2

Aplicacin a la existencia de irracionales


Ejemplo 3.5. Demostrar que existe

5.

Prueba: Me estn pidiendo encontrar un nmero real x R, de modo que x2 = 5.


Ahora considerando f (x) = x2 5 la cual es una funcin continua en [2, 3], y se cumple
con
f (2) = 1 < 0 < 4 = f (3)
Luego por el teorema del cero, existe x h2, 3i de modo que f (x) = 0, es decir
x2 = 5

Ejemplo 3.6. Demostrar que existe un nmero real

7.

Prueba: Se deja al estudiante, para que imite la demostracin anterior.

3.1.

Continuidad y los Intervalos

El presente resultado es especial ya que teniendo una funcin continua, la imagen de


un intervalo es tambin un intervalo, aunque no necesariamente del mismo tipo. En la
demostracin del siguiente teorema haremos uso de la caracterizacin de los intervalos,
la cual es una forma de definir los intervalos, recordemos que: I R no vaco, I es un

74

CAPTULO 3. TEOREMAS FUNDAMENTALES DE CONTINUIDAD

intervalo si y solo si x, y I con x < y, implique (x, y) I.


Este es un resultado muy bueno para poder trabajar con intervalos, cualquiera que sea, se
debe tomar un par x, y de nmero reales en I, tal que x < y, luego considerar w (x, y),
es decir x < w < y, para luego, si es que I es un intervalo poder demostrar que
wI
Debemos recordar que existen nueve tipos de intervalos, las cuales estn distribuidos en
dos grupos:
Primer grupo, llamados tambin intervalos finitos o acotados : [a, b], (a, b), [a, b) y (a, b].
Segundo

grupo,

conocidos

como

los

intervalos

infinitos

no

acotados:

[a, +), (a, +), (, b], (, b) y (, +).


En algunos casos para efecto de un proceso de demostracin se considera el intervalo
degenerado [a, b] con a = b, la cual es un conjunto unitario y (a, b) con b < a, que resulta
el conjunto vaco, las cuales son llamados como se dijo intervalos degenerados, pero son
casos muy especiales para algunas demostraciones muy particulares.
Teorema 3.9. Sea f una funcin continua en un intervalo I, entonces f (I) es tambin
un intervalo.
Prueba: En el caso especial que f (I) sea un conjunto unitario {a}, ya se tiene por
cumplido con ser un intervalo, aunque sea un intervalo degenerado, como caso especial,
[a, a], luego se verifica con lo pedido.
En otro caso, el cual es el general, hay que probar que f (I) sea un intervalo, para ello
tomemos dos elementos de l
w1 , w2 f (I)
con w1 < w2 . Sea w (w1 , w2 ), tenemos que probar que w f (I).
Como w1 , w2 f (I), debe existir a, b I tal que
w1 = f (a), w2 = f (b)
note que a 6= b, luego sin prdida de generalidad podemos considerar a < b, entonces
(a, b) I, por otro lado como a, b I, luego tenemos que
[a, b] I

3.1. CONTINUIDAD Y LOS INTERVALOS

75

como f es continua en [a, b], por otro lado se tiene w1 < w2 , entonces f (w1 ) < f (w2 ),
como w (w1 , w2 ), por lo tanto tenemos
f (a) < w < f (b)
por lo tanto existe c (a, b) tal que f (c) = w, entonces c I de modo que se cumpla con
f (c) = w, por lo tanto
w f (I)

Observacin: Si consideremos f : (0, 1) (2, 3) R de modo que

1 , x (0, 1)
f (x) =

2 , x (2, 3)
es claro que f es continua en (0, 1) (2, 3) pero su imagen no es un intervalo, ya que
f ((0, 1) (2, 3)) = {1, 2}.
Por otro lado consideremos
f (x) =

1 , x Q (0, 1)

2 , x Q (0, 1)

cuyo dominio es el intervalo I = (0, 1), pero la imagen f (I) no lo es, esto es porqu f no
es continua.
Tomemos ahora la funcin cuadrtica f (x) = x2 , x I. Tomemos I = (1, 1), luego
f (I) = [0, 1) es notorio que el intervalo resultante, no es del mismo tipo que el de I. Por
lo tanto la continuidad no asegura que preserve el mismo tipo de intervalo.
Teorema 3.10. Si f es continua en [a, b], entonces f es una funcin acotada en [a, b], es
decir existe k > 0 tal que
|f (x)| k, x [a, b]
Prueba: Como f es continua en [a, b], se tiene que |f | es tambin continua en [a, b].
Supngase que |f | no est acotada superiormente, entonces para cada r > 0, existe x1
[a, b] tal que
r < |f (x1 )|

76

CAPTULO 3. TEOREMAS FUNDAMENTALES DE CONTINUIDAD

por lo tanto, para cada n N, existe xn [a, b] de modo que


n < |f (xn )|

(3.1)

es decir, se ha construido una sucesin (xn ) en [a, b] y (|f (xn )|) la sucesin de las imgenes,
de modo que se cumple con
lm |f (xn )| = +

n+

(3.2)

Ahora como la sucesin (xn ) se encuentra en el intervalo compacto [a, b], entonces existe
una subsucesin (xni ) que sea convergente, este resultado es uno de los buenos teoremas
de sucesiones sobre compactos, en esta parte simplemente lo utilizaremos, por lo tanto
lm xni = L [a, b]

i+

luego como |f | es continua en L, se tiene que


lm |f (xni )| = f (L)
lo cual es imposible por (3.2).
Observacin: Uno de los buenos resultados es, que toda sucesin acotada (xn ), posee
una subsucesin convergente.
Teorema 3.11. Demostrar que si f es continua en [a, b], entonces existe x1 [a, b] tal
que f (x1 ) = nf [a,b] (f ).
Prueba: Estamos utilizando la siguiente notacin
nf (f ) = nf{f (x) : x [a, b]}
[a,b]

por otro lado, como f es una funcin continua en [a, b], entonces f est acotada en [a, b], de
manera particular, est acotada inferiormente, luego existe c R tal que c f (x), x
[a, b], luego se encuentra garantizada la existencia del nfimo
nf (f ) = m
[a,b]

Supongamos que m < f (x), x [a, b], note esta tcnica de desmostracin, ya que queremos que se de la igualdad, consideramos el caso contrario, luego podemos crear una
funcin g definida por
g(x) =

1
f (x) m

77

3.1. CONTINUIDAD Y LOS INTERVALOS

continua en [a, b], la cual es es acotada, por ser continua sobre [a, b], por lo tanto es acotada
superiormente, esto es
|g(x)| K1 , x [a, b], K1 > 0
por lo tanto g(x) K1 , es decir
1
K1
f (x) m
por lo tanto

1
K1

f (x) m es decir
m+

luego m +

1
K1

1
f (x), x [a, b]
K1

es una cota inferior por lo tanto


m+

luego

1
K1

1
m
K1

0, la cual no se puede dar.

Suficiente con ello para haber obtenido una contradiccin y al mismo tiempo haber demostrado lo pedido.
Teorema 3.12. Sabiendo que si f es continua en [a, b], entonces existe x2 [a, b] tal que
f (x2 ) = sup(f )
[a,b]

Prueba: Como la funcin f es continua en el intervalo [a, b], entonces f es una funcin
acotada en [a, b], de manera especial f esta acotada superiormente, luego existe
sup(f ) = s
[a,b]

Supngases que para todo x [a, b], f (x) < s.


Con ello podemos considerar la siguiente funcin
g(x) =

1
, x [a, b]
s f (x)

entonces g es acotada luego existe k2 > 0 tal que |g(x)| < k2 , luego
1
< k2
s f (x)
por lo tanto

1
k2

< s f (x), es decir f (x) < s k12 , x [a, b] por lo tanto existe x2 [a, b]

tal que existe x2 [a, b] tal que


f (x2 ) = s.

78

CAPTULO 3. TEOREMAS FUNDAMENTALES DE CONTINUIDAD

Teorema 3.13. Sea f una funcin continua en [a, b], entonces f ([a, b]) = [c, d], para algn
c, d R.
Prueba: Como f es una funcin continua sobre el compacto [a, b], entonces existe x1
[a, b] tal que
f (x1 ) f (x), x [a, b]
tambin se tiene la existencia de un x2 [a, b] tal que
f (x) f (x2 ), x [a, b]
por lo anterior podemos afirmar lo siguiente.
Afirmamos: f ([a, b]) = [c, d] donde c = f (x1 ), d = f (x2 ).
Demostrando:
Sea w f ([a, b]) un elemento cualquiera, luego existe x0 [a, b] tal que w = f (x0 ) pero
por las ecuaciones anteriores tenemos
f (x1 ) f (x0 ) f (x2 )
es decir c , w d, es decir w [c, d].
Sea w [c, d] un elemento arbitrario luego
cwd
es decir f (x1 ) w f (x2 ) luego tenemos
Caso 1: si w = f (x1 ) o w = f (x2 ), entonces w f ([a, b]).
Caso 2: si f (x1 ) < w < f (x2 ) como f es continua en [x1 , x2 ] o [x2 , x1 ], entonces tenemos
que existe c < x1 , x2 > o c < x2 , x1 > tal que se cumpla con
f (c) = w
donde c [a, b], por lo tanto w f ([a, b]).
Observacin:
Considere f (x) = x2 , x R, luego tenemos
f ([1, 1]) = [0, 1]

3.1. CONTINUIDAD Y LOS INTERVALOS


tambin
f ([0, 4]) = [0, 16]

79

4
Problemas Resueltos
Ejemplo 4.1. Considere la siguiente funcin f (x) =

x2 x2

, x 6= 2

,x = 2

x2

. Determine si f

es continua en x = 2.
Prueba: Lo demostraremos por medio de lmite, calculemos
lm f (x) = 3

x2

y f (2) = 3, por lo tanto lmx2 f (x) = f (2), con lo cual f es una funcin continua en
x = 2.
Observacin:
Ntece que el modelo de resolucin, ha sido por medio de lmite, teniendo presente que 2
es un punto de acumulacin del dominio de f , puede tambin se pudo haber resuelto por
deficin de continuidad.
Tambin notar el estudiante que la funcin f es continua en todo su dominio.
Ejemplo 4.2.
Prueba:
Ejemplo 4.3. Considere la funcin f (x) =

,x Q

. Pruebe que f es nicamente

x , x I
continua en x0 = 0.
Prueba: Sea x0 R dado arbitrariamente, en este punto dividamos el problema en dos:
si x0 6= 0;
81

82

CAPTULO 4. PROBLEMAS RESUELTOS


si x0 = 0.

Caso 1: si x0 6= 0, demostremos que f no es continua en x0 . Una forma de ver ello es


aplicando lmites, utilizamos lo que hemos venido realizando, por densidad, tanto de los
racionales como de los irracionales, respecto a los reales, existe dos sucesiones (xn ) en Q
y (yn ) en I tal que
lm xn = x0 , lm yn = x0

(4.1)

supongamos que f sea continua en x0 , de la ecuacin (4.1) tenemos


lm f (xn ) = f (x0 ) , lm f (yn ) = f (x0 )
pero por definicin de la funcin f , tenemos f (xn ) = xn y f (yn ) = yn , con lo cual
tenemos
lm xn = f (x0 ) , lm(yn ) = f (x0 )
es decir
x0 = f (x0 ), x0 = f (x0 )
por lo tanto x0 = x0 , es decir x0 = 0, pero esto es imposible, luego se ha encontrado un
hecho contradictor, por lo tanto tenemos que negar lo supuesto inicialmente, es decir f
no es continua en x0 .
Caso 2: si x0 = 0, probaremos que f si es continua en dicho punto, veamos el proceso
o la tcnica de la demostracin, en esta oportunidad utilizaremos la definicin.
Sea  > 0 cualquiera, podemos considerar = , si x Dom(x) = R y |x 0| < ,
entonces
|x| < 
Por otro lado calculemos
|f (x) f (0)| = |f (x) 0| = |f (x)|
sobre el valor de x podemos desprender dos subcasos:
Si x Q , entonces f (x) = x, por lo tanto
|f (x) f (0)| = |f (x)| = |x| < 

83
Si x I, tenemos f (x) = x, es decir
|f (x) f (0)| = |f (x)| = | x| = |x| < 

Ejemplo 4.4. Considere la siguiente funcin llamada funcin signo, definida por

|x| , x 6= 0
x
sgn(x) =

0 , x = 0
Pruebe que f es continua en cada punto, excepto en x = 0.
Prueba: Sea x0 R con x0 6= 0, luego tenemos dos posibilidades
si x0 < 0;
si x0 > 0.
Caso 1: si x0 < 0, sea  > 0 arbitrario, podemos considerar = |x0 | = x0 > 0. Ahora
tomando
x R, |x x0 | < |x x0 | < x0
por lo tanto x0 < x x0 < x0 , es decir 2x0 < x < 0, ahora como x 6= 0, se tiene que
sgn(x) =

|x|
x

= 1 para nuestro caso


sgn(x0 ) =

|x0 |
= 1
x0

calculemos
|sgn(x) sgn(x0 )| = | 1 (1)| = 0 < 
Caso 2: si x0 > 0 realizar un procedimiento muy parecido a lo anterior.
Finalmente podemos indicar que sgn es una funcin continua en x, x 6= 0.
Afirmamos que sgn es discontinua en x = 0.
una forma de demostrar es por medio de lmites, ya que x = 0 es un punto de acumulacin
del Dom(f ).
Calculemos

84

CAPTULO 4. PROBLEMAS RESUELTOS


lmx0+ sgn(x) = 1;
lmx0 sgn(x) = 1.

lo anterior me est indicando que no existe lmx0 sgn(x) por lo tanto dicha funcin es
discontinua en 0.
Observacin: Note que en el proceso de demostracin se ha dividido en caso, de
este modo logramos vencer el problema, tambin la importancia de la equivalencia entre
continuidad y lmites, fue clave para el problema.
Ejemplo 4.5. Supngase que f : A R es continua en A y f (x) = 0 para todo x en un
subconjunto denso en A.
Pruebe que x A, f (x) = 0.
Prueba: Sea x0 A arbitrario, considere D A donde D es denso en A, del problema
existe (xn ) en D tal que
lm xn = x0
ahora tenemos por continuidad en x0 que
lm f (xn ) = f (x0 )
por lo tanto, por hiptesis f (xn ) = 0, es decir f (x0 ) = 0.
Ejemplo 4.6. Sea f : A R una funcin continua en A y f (x) = 0 para todo x Q A,
con Q A es denso en A. Pruebe que x A, f (x) = 0.
Prueba: Sea a A arbitrario, ahora por densidad, existe una sucesin (xn ) en Q A tal
que lm xn = a, entonces por continuidad de f en a, se tiene
lm f (xn ) = f (a)
ahora como f (xn ) = 0, luego f (a) = 0.
Debemos recordar que, si X R, luego X es denso en R para cada a, b R, a < b
existe x X tal que
a<x<b

85
la cual es equivalente, en trmino de sucesiones: sea X R, X es denso en R para
cada a R, existe (xn ) en X tal que
lm xn = a
Tambin tenemos la densidad relativa: sea X R, A X. A es denso respecto X
para cada x X, existe (an ) en A tal que
lm an = x
Ejemplo 4.7. Dar ejemplos para cada funcin en los siguientes casos:
1. funciones f, g : R R discontinuas en cada punto de R, pero f + g es continua
para todo los reales.
2. funciones f, g : R R discontinuas en cada punto de R, con f g continua en todo
R.
Prueba:

1. Basta dar las funciones en forma explcita o concreta

1
,x Q
f (x) =

1 , x I

tambin
g(x) =

1 , x Q
,x I

se nota que f + g es una funcin constante y nula, luego continua en todo R.


2. considere
f (x) = g(x) =

,x Q

1 , x I
por lo tanto (f g)(x) = f (x)g(x) = 1, x R.

Ejemplo 4.8. Determine los intervalos en R, en los cuales las siguientes funciones sean
continuas

86

CAPTULO 4. PROBLEMAS RESUELTOS


1. f (x) =

3x+5
.
x2 4

2. f (x) = sen( x+2


).
x2
3. f (x) = cos
4. f (x) =

x.

cos x.

5. f (x) = tan x.
6. f (x) = tan(sen x).
1. R \ {2, 2}.

Prueba:

2. R \ {2}, ya que la funxin seno es continua en todo los reales.


3. [0, +).
4.

nZ [

+ 2n, + 2n].

5. R \ {(2n + 1) 2 : n Z}.
6. como 2 < 1 sen x 1 < 2 , entonces f es continua en todo los reales.

Ejemplo 4.9. Determine los intervalos sobre los cuales las siguientes funciones son continuas
(a) f (x) =
(b) f (x) =
(c) f (x) =
(d) f (x) =
Prueba:

x2 + 5.
x2

x1
.
+ 2x 3

3x 2.
x2

x
.
+1

(a) Una forma de encontrar lo pedido, es utilizar la relacin entre la con-

tinuidad y las funciones compuestas.


Considere las siguientes funciones
g(x) =

x, x 0

87
h(x) = x2 + 5, x R
se tiene luego f (x) = g h(x), x R, como h es continua en todo R, por ser funcin
polinomial y g es continua en h(x), luego f es continua en todo x R.
(b) considere g(x) = x 1 y h(x) = x2 + 2x 3 = (x + 3)(x 1) y f (x) =

g(x)
,
h(x)

note

que g es continua en D1 = R. Luego evitando que h(x) = 0, se tiene h es continua


no nula en D2 = R \ {1, 3}, luego por propiedad
f (x) =

g(x)
h(x)

es una funcin continua en D1 D2 = R \ {1, 3}.


(c) Consideremos para este caso
g(x) =

h(x) = 3x 2
luego f (x) = g h(x), x 23 , luego h es continua en x0 [ 23 , +) y g es continua
en h(x0 ), por lo tanto f es continua en
2
x0 [ , +)
3
(d) Se verifica que f es continua en todo los reales.

Ejemplo 4.10. Dada la siguiente funcin f (x) =

x3 + 2x2 + x

1. f es continua en x = 1?
2. f es continua en x = 0?
3. lmx1 f (x) = f (1)?
4. Calcule lmx0 f (x).
Prueba: Lo primero que debemos hacer, es calcular el dominio de dicha funcin, para
ello podemos considerar
x3 + 2x2 + x 0
x(x2 + 2x + 1) 0
x(x + 1)2 0

88

CAPTULO 4. PROBLEMAS RESUELTOS

por lo tanto tenemos que


x = 1 o x 0
es decir Dom(f ) = {1} [0; + >.

1. De lo anterior tenemos, como x = 1 es un punto aislado del dominio de la funcin


f , tenemos que f es continua en 1.
2. Una manera de ver este problema, es por composicin de funciones considerando

g(x) = x y h(x) = x3 + 2x2 + x, por lo tanto


f (x) = g h(x), x Dom(f )
ahora bien, tenemos que g es una funcin continua en h(0) = 0 y h es continua en
x = 0, por lo tanto f es continua en 0.
3. Al ser x = 1 un punto aislado, no tiene sentido expresar lm x 1f (x), luego
podemos decir que es falso.
4. Se deja al estudiante para que detalle su respuesta.

Ejemplo 4.11. Usando la definicin de continuidad pruebe que la funcin f (x) = 2x2 +
3x + 5 es continua en x0 = 1.
Prueba: En la primera parte, daremos una exploracin para determinar el > 0 adecuado, para luego dar la demostracin en forma precisa.
Primera Parte:
calculemos
|f (x) f (x0 )| = |2x2 + 3x + 5 4| = |2x2 + 3x + 1| = |2x + 1||x + 1|

(4.2)

en la ecuacin anterior vemos la primera dificultad, es diferente si nos hubiera resultado


por ejemplo 3|x + 1|, para poder transformarlo adecuadamente, tenemos que considerar
un =

1
2

> 0 (es un intento), es decir


|x x0 | = |x + 1| < 1 =

1
2

89
por lo tanto
|x + 1| <

1
2

de aqu se tiene que |2x + 1| < 2, juntando ambos resultados tenemos


|f (x) f (x0 )| = |2x + 1||x + 1| < 2|x + 1| < 
es decir |x + 1| <


2

considerando as, un segundo delta


2 =


2

por lo tanto podemos considerar = mn{1 , 2 } > 0. Porqu no es correcto considerar


= max{1 , 2 }?
Segunda Parte:
Sea  > 0 tomado arbitrariamente, luego consideremos = mn{ 21 , 2 }, ahora tomemos
x Dom(f ) = R, |x x0 | < |x + 1| <
por lo tanto
|x + 1| <

1
2

de aqu se obtiene
|2x + 1| < 2

(4.3)

tambin se tiene |x + 1| < 2 , por lo tanto


|x + 1| <


2

ahora s podemos realizar el siguiente clculo


|f (x) f (x0 )| = |2x2 + 3x + 1| = |2x + 1||x + 1| < 2|2x + 1| < 
listo, esto nos demuestra que f es continua en 1.
Ejemplo 4.12. Pruebe que la funcin f (x) = 4x2 5x 3 es continua en x = 2.

(4.4)

90

CAPTULO 4. PROBLEMAS RESUELTOS

Prueba: Sea  > 0 dado arbitrariamente, luego podemos considerar


= mn{1,


}
15

ahora, sea x Dom(f ) = R, |x x0 | < , por lo tanto |x 2| < 1, es decir


(4.5)

|4x + 3| < 15
luego como |x x0 | <


,
15

implica que
|x 2| <


15

(4.6)

por lo tanto podemos calcular


|f (x) f (x0 )| = |4x2 5x 6| = |4x + 3||x 2| < 15|x 2| < 

Nota: est claro que antes de este desarrollo, se ha realizado un proceso para descubrir
el adecuado.
Ejemplo 4.13. Sea f (x) =

sen( 1 ) , x 6= 0
x

y g(x) =

x sen( 1 ) , x 6= 0
x

,x = 0
,x = 0
Determine cual o cuales de las funciones anteriores son continuas en x = 0. Justifique su
respuesta.
Prueba: Con respecto a la funcin f :
Vamos a aplicar la equivalencia de continuidad, con lmites, ya que 0 es un punto de
acumulacin (p.a) del dominio
f es continua en x0 = 0 lm f (x) = f (0)
x0

Aqu el problema es que dicho lmite no existe, para ello basta tomar dos sucesiones que
converjan al cero y las sucesiones de las imgenes no existan al menos en una de ellas o
no converjan al mismo valor.
Considere
xn =

1
1
, yn =
, n N
2n
(4n + 1) 2

91
es claro que se cumple con
lm xn = 0, lm yn = 0
pero
lm f (xn ) = 0, lm f (yn ) = 1
por lo tanto f no es continua en x0 = 0.
Con respecto a la funcin g:
tomando lmite y aplicando la propiedad adecuada tenemos
1
lm g(x) = lm x sen( ) = 0
x0
x0
x
entonces tenemos lmx0 g(x) = g(0), por lo tanto tenemos que g es continua en 0.
Ejemplo 4.14. Demostrar que la ecuacin x = a sen x + b, 0 < a < 1, b > 0 tiene una
raz x0 tal que
0 < x0 a + b.
Prueba: Primero consideremos la siguiente funcin f (x) = x a sen x b, x [0, a + b],
por lo tanto tenemos que f es continua en [0, a+b]. En este punto dividiremos el problema
en dos partes:
Si sen(a + b) = 1, entonces podemos tomar x0 = a + b, la cual es la raz que nos estn
pidiendo.
Si sen(a + b) 6= 1, entonces podemos calcular
f (0) = 0 a sen 0 b = b < 0
Tambin calculemos f (a + b) = a + b a sen(a + b) b = a a sen(a + b) > 0.
Por lo tanto tenemos que f (0) < 0 < f (a + b), luego por el teorema del valor intermedio
(T.V.I), nos garantiza la existencia de
x0 < 0, a + b >
de modo que f (x0 ) = 0, es decir x0 a sen x0 b = 0, por lo tanto
x0 = a sen x0 + b

92

CAPTULO 4. PROBLEMAS RESUELTOS

donde x0 es una raz con 0 < x0 < a + b.


Finalmente de ambos casos tenemos la existencia de tal ran pedida.
Observacin:
Note la importancia de dividir el problema en dos partes, y que en cada parte dividida se
debe buscar con lo que se pide.
En este problema el T.V.I ha sido clave para su resolucin. Este ejemplo muestra la
relacin entre la continuidad de una funcin y las ecuaciones.
Ejemplo 4.15. Sean f y g dos funciones continuas en c. Demostrar que si
1. f (c) > 0, entonces existe un > 0 de modo que f (x) > 0 para todo x (c, c+).
2. f (c) < 0, entonces existe un > 0 tal que f (x) < 0 para todo x (c , c + ).
3. f (c) < g(c), entonces existe un > 0 tal que f (x) < g(x) para todo x (c, c+).
Prueba:

1. Considere  = f (c) > 0, ahora como f es continua en el punto c, deber

existir un > 0 de modo que: x Dom(f ), |x c| < , implica


|f (x) f (c)| <  = f (c)
por lo tanto f (c) < f (x) f (c) < f (c), por lo tanto tenemos f (x) > 0, es decir:
si x (c , c + ), se cumple
f (x) > 0
2. Para este caso tomaremos  = f (c) > 0 y como f es continua en c, tenemos la
existencia > 0 tal que x Dom(f ) y |x c| < , esto implica |f (x) f (c)| <  =
f (c), implica f (x) < 0.
3. Basta tomar  =

g(c)f (c)
.
2

Ejemplo 4.16. Considere la siguiente funcin f (x) =

3 + x , x 1

3 x , x > 1

puntos de discontinuidad para f .

. Encontrar los

93
Prueba: Observando la funcin dada podemos intuir que:
Afirmacin 1: f es discontinua en x = 1.
Ahora como x = 1 es un punto de acumulacin del dominio de la funcin f (Dom(f ) = R),
basta luego ver su lmite, veamos ello
lm f (x) = lm (3 x) = 2
x1

x1+

lm f (x) = lm (3 + x) = 4

x1

x1

con lo cual tenemos que no existe lmx1 f (x), es decir f no es continua en x = 1.


Afirmacin 2: f es continua en x0 , x0 6= 1.
En efecto, sea x0 6= 1 arbitrario. Si x0 > 1 se tiene
lm f (x) = lm (3 x) = 3 x0 = f (x0 )

xx0

xx0

luego por propiedad f es continua en x0 . En el caso que x0 < 1 igual, por lo tanto f es
continua en f .
Ejemplo 4.17. Pruebe que

sen x
1
= , existe una raz entre 0 y .
x
2

Prueba: Para resolver este tipo de problemas, es necesario crear una funcin, para este
caso tomando
g(x) = 2 sen x x
la cual es continua sobre [0, ] en particular sobre el intervalo [ 2 , ].
Ahora como

g( ) = 2 sen = 2 > 0
2
2
2
2
tambin tenemos la siguiente desigualdad
g() = 2 sen = < 0
por lo tanto, juntando ambas inecuaciones

g() < 0 < g( )


2
por lo tanto tenemos la existencia x ( 2 , ) tal que
g(x) = 0

94

CAPTULO 4. PROBLEMAS RESUELTOS

por lo tanto 2 sen x x = 0, es decir 2 sen x = x, como x 6= 0, tenemos


1
sen x
=
x
2

Ejemplo 4.18. Sea f la funcin mximo entero de x,


(i) f es discontinua en x0 , x0 Z.
(ii) f es continua en x0 , x0 6 Z.
Prueba:

(i) Sea x0 Z un entero cualquiera, debido a que dicho punto es de acu-

mulacin respecto al dominio de f , entonces podemos enfrentar la continuidad en


trminos de lmites, de manera especial los laterales
lm f (x) =

xx+
0

lm

xx+
0

con lo cual tenemos la no existencia de tal lmite.


(ii) Detallar.

Observacin:
Creemos que no es vital la presencia de una grfica, pero definitivamente ayuda a entender
graficamente lo sucedido.
Ejemplo 4.19. Sea f una funcin real de variable real, tal que
|f (x)| |x|, x R.
Pruebe que f es continua en x = 0.
Prueba: Sea  > 0 arbitrario, tomemos =  > 0. Si x R, con |x 0| < , entonces
|x| < =  |x| < 
Finalmente podemos calcular
|f (x) f (0)| = |f (x)| |x| < .

95
Observacin:
Note la importancia de tomar el como el mismo valor de , la cual soluciona nuestro
problema.
Ejemplo 4.20. Determine los puntos de discontinuidad de la siguiente funcin
f (x) =

x2 9
x2 2x 3

e indicar el tipo de discontinuidad de dichos puntos.


Prueba: Lo primero que se va a realizar, es factorizar la expresin dada
(x + 3)(x 3)
x+3
2
x2 9
=
=
=1+
, x R\\{3, 1}
f (x) = 2
x 2x 3
(x 3)(x + 1)
x+1
x+1
por lo tanto se tiene que f es discontinua en 1 (es la cual hay una asntota vertical,
luego es del tipo discontinua irremovible).
En cambio en 3 es discontinuidad removible.

Ejemplo 4.21. Sean f, g : R R dos funciones continuas, tales que f (x) < g(x), x R.
Pruebe que existe una funcin h : R R continua, tal que
f (x) < 2h(x) < g(x), x R.
f (x) + g(x)
, x R, ahora debido a la continuidad de f
4
y g se deduce la continuidad de la funcin h.
Prueba: Basta tomar h(x) =

Tambin tenemos por cierto que


f (x) <

f (x) + g(x)
< g(x)
2

es decir f (x) < 2( f (x)+g(x)


) < g(x), por lo tanto
4
f (x) < 2h(x) < g(x).

Ejemplo 4.22. Muestre que para cada d (3, 5), existe c (0, 1) tal que
c8 + c5 c4 + c + 3 = d

96

CAPTULO 4. PROBLEMAS RESUELTOS

Prueba: la tcnica es formar una funcin con el dato dado, de la siguiente manera
f (x) = x8 + x5 x4 + x + 3 d
la cual es continua en [0, 1]. Calculemos primero
f (0) = 3 d < 0
luego
f (1) = 5 d > 0
por lo tanto tenemos f (0) < 0 < f (1), luego por el T.V.M para funciones continuas se
tiene la existencia de c (0, 1) de modo que
f (c) = 0
es decir c8 + c5 c4 + c + 3 d = 0, luego
c8 + c5 c4 + c + 3 = d

Observacin:
Note la importancia de comenzar la demostracin formando una funcin especial, que se
deriva de la informacin brindada.
Ejemplo 4.23. Usando el teorema del valor intermedio(T.V.I) probar que la funcin
polinomial f (x) = x3 + 2x 1 posee al menos una raz real.
Prueba: Como f (x) es una funcin polinomial (de grado 3), es una funcin continua
sobre todo los nmeros reales, de manera especial en el intervalo [0, 1] y por otro lado
tenemos el siguiente resultado
f (0) = 1 < 0 < 2 = f (1)
luego tenemos la condiciones para poder utilizar el T.V.I, por lo tanto existe c (0, 1) de
modo que se cumpla con
f (c) = 0
es decir c3 + 2c 1 = 0 es decir c es una raz de f (x) = 0.

97
Observacin:
Si uno realiza la grfica de dicha funcin, la cual puede ser por medio de los graficadores
Geogebra o Wolfram Alpha, notara que la curva solo corta al eje de las abscisas en un solo
punto x = c, esto significa que las otras races que posee son complejas: una conjugada
de la otra.
Ejemplo 4.24. Considere la siguiente funcin

x 3
,x 2
f (x) =

ax2 , x > 2
Determine el valor de a, de modo que f sea una funcin continua en todo su dominio.
Prueba: Como f es continua en R, de manera muy particular, es continua en 2. Ahora
como x = 2 es un punto de acumulacin del dominio de f , para ello utilizaremos la
equivalencia con lmites, es decir
lm f (x) = lm f (x) = f (2)

x2+

x2

calculamos
lm f (x) = lm+ (ax2 ) = 4a

x2+

x2

tambin
lm f (x) = lm (x3 ) = 8

x2

x2

por lo tanto
4a = 8
luego a = 2.
Ejemplo 4.25. Hallar los valores de a y b de modo que la funcin siguiente, sea continua
en toda la recta real
f (x) =

x + 1

,1 < x < 3

x2 + bx + c , |x 2| 1
Prueba: Tenemos que reformular la funcin dada
|x 2| 1 x 2 1 x 2 1
x3x1

98
por lo tanto tenemos

CAPTULO 4. PROBLEMAS RESUELTOS

x2 + bx + c , x 1

f (x) = x + 1
,1 < x < 3

x2 + bx + c , 3 x

Debido a que f es continua en x = 1, se tiene


lm f (x) = lm+ f (x) = f (1)

x1

x1

se tiene luego 1 + b + c = 2, luego b + c = 1.


Tambin se tiene la continuidad en x = 3, luego
4 = 9 + 3b + c
luego 3b + c = 5, por lo tanto resolviendo las ecuaciones anteriores, tenemos
b = 3, c = 4.

Ejemplo 4.26. Sea f : R R una funcin que satisface con


|f (x)| |x|, x R.
Probar que f es continua en x = 0.
Prueba: Sea  > 0 tomado arbitrariamente, tenemos que encontrar un > 0 adecuado,consideremos
=>0
es claro, que esta intuicin no proviene de forma no significativa, sino se ha trabajado
previamente para deducir ello.
Veamos si verifica con la defincin de continuidad, considere x R, |x 0| < , luego
|x| < 
Finalmente,calculemos
|f (x) f (0)| = |f (x)| |x| < 
basta con ello, para haber demostrado que la funcin f es continua en x = 0.

99
Observacines:
Note que en la hiptesis |f (x)| |x|, x R, como me dejan libertad por medio del
cuantificador universal (), tomemos en particular el valor de x = 0, luego tendramos
|f (0)| |0|
es decir |f (0)| 0, por lo tanto f (0) = 0.
Tambin podemos considerar el siguiente argumento para solucionar el problema anterior:
debido a que el dominio de f es todo R, luego x = 0 es un punto de acumulacin del
Dom(f ), por lo tanto, la defincin de continuidad tiene un equivalente en lmite
f es continua en 0 lm f (x) = f (0)
x0

luego notando que


0 |f (x)| |x|
aplicando lmites a las inecuacin anterior obtenemos lmx0 f (x) = 0 = f (0), por lo
tanto f es una funcin continua en x = 0.
Ejemplo 4.27. Dar una funcin f : R R de modo que no sea continua en nign punto
de su dominio.
Prueba: Basta considerar la funcin conocida

1 , x Q
f (x) =

0 , x I
Ahora podemos afirmar que f es discontinua en x0 , x0 R.
En efecto, supngase que f es continua en x0 , es decir estamos utilizando para su demostracin, el mtodo indirecto. Por lo tanto existe 1 > 0 tal que
x R, |x x0 | < 1 |f (x) f (x0 )| <

1
4

ahora por densidad de Q en R existe q Q tal que |q x0 | < 1 , por lo tanto de la


ecuacin anterior tenemos
|f (q) f (x0 )| <

1
1
|1 f (x0 )| <
4
4

100

CAPTULO 4. PROBLEMAS RESUELTOS

tambin podemos decir, por la densidad de los irracionales en R, que existe r I de modo
que
|r x0 | < 1
por lo tanto tenemos |f (r) f (x0 )| < 14 , luego |0 f (x0 )| < 41 , luego
|f (x0 )| <

1
4

de las inecuaciones anteriores tenemos


|1| = |1 f (x0 ) + f (x0 )| |1 f (x0 )| + |f (x0 )| <

1 1
1
+ =
4 4
2

lo cual es imposible.
Por lo tanto f es discontinua en toda la recta real.
Ejemplo 4.28. Encontrar una funcin f que sea continua nicamente en tres puntos de
su dominio.
Prueba: Como me dejan libertad de colocar el dominio adecuado, basta con considerar
X = {1, 2, 3}, f : X R tal que
f (1) = 1, f (2) = 2, f (3) = 3
Debido a que el dominio es un conjunto finito, luego f es continua en todo su dominio, la
cual cumple con las condiciones pedida.
Observacin:
Agreguemos una condicin mas al problema, de la siguiente manera: Encontrar una funcin f con dominio todo los reales R,de modo que sea continua nicamente en tres puntos
de su dominio.
Para este problema la dificultad ha aumentado,es claro grficamente que si existe tal
funcin, se deja al estudiante encontrar tal funcin.
Ejemplo 4.29. Supngase que g es una funcin continua en x = 0 y g(0) = 0. Probar
que f es continua en 0, siempre que x Dom(f ) = Dom(g), |f (x)| |g(x)|.
Prueba: Considere  > 0 arbitrario, ahora como g es continua en 0, existe > 0 tal que
x Dom(g), |x 0| < |g(x) g(0)| < 

101
por lo tanto tenemos
x Dom(g) |x| < |g(x)| < 

(4.7)

ahora, se ha encontrado el > 0 adecuado, veamos que el, sea el adecuado valor que
buscamos: Sea x Dom(f ) y |x 0| < por lo tanto x Dom(g) y |x| < , entonces
por la ecuacin (4.7) obtenemos
|g(x)| < 
Luego finalmente, calculemos |f (x) f (0)| = |f (x) 0| = |f (x)| |g(x)| < .
Ejemplo 4.30. Dar un ejemplo de una funcin f que no sea continua en ningn punto,
pero |f | es continua en todos los puntos.
Prueba: Basta considerar la siguiente funcin f , definida por

1
,x Q
f (x) =

1 , x I
se tiene por verdadero, que f es una funcin discontinua en todo su dominio, pero al
considerar la funcin |f |, obtenemos
x R, f (x) = 1
la cual es una funcin constante, con la caracterstica que es continua en todo su dominio.

Ejemplo 4.31. Considere f : R R una funcin que satisface la siguiente propiedad


x, y R, f (x + y) = f (x) + f (y)
y f es continua en x = 0. Demostrar que f es una funcin continua en todo su dominio.
Prueba: Sea a R un real cualquiera, podemos atacar el problema de la continuidad
por medio de la teora de lmites, y esto es posible, ya que a es un punto de acumulacin
del dominio de f , luego tenemos que verificar
lm f (x) = f (a)

xa

102

CAPTULO 4. PROBLEMAS RESUELTOS

comencemos el proceso del clculo


lm f (x) = lm f (a + h)

xa

h0

= lm [f (a) + f (h)]
h0

= f (a) + lm f (h)
h0

= f (a) + f (0)
= f (a + 0) = f (a)
Listo, con ello se ha probado lo pedido.
Ejemplo 4.32. Probar que si f es una funcin continua en a, entonces |f | es continua
en a.
Prueba: Sea  > 0 arbitrario, debido a que f es continua en a, entonces existe 1 > 0 tal
que
x Dom(f ) |x a| < 1 |f (x) f (a)| < 

(4.8)

Luego podemos considerar el mismo 1 > 0 encontrado en la ecuacin (4.8).


Sea x Dom(f ), |xa| < 1 , entonces x Dom(f ), |xa| < 1 . Luego para estos valores
de x, calculemos
||f |(x) |f |(a)| = ||f (x)| |f (a)|| |f (x) f (a)| < 

Ejemplo 4.33. Sea f : R R una funcin continua. Demostrar que se puede escribir de
la forma
f =g+h
con g, h : R R, donde g es funcin continua par y f funcin impar continua.
Prueba: La clave de este ejercicio es expresar f (x) de la siguiente manera
f (x) =
considerando g(x) =

f (x)+f (x)
2

par y continua, veamos ello:

f (x) + f (x) f (x) f (x)


+
2
2
y h(x) =

f (x)f (x)
,
2

solamente falta confirmar que g es

103
Debido a que f (x) puede ser vista como la composicin de dos funciones continuas
f (I)
por lo tanto f (x) es continua, entonces g es continua.
Calculemos
g(x) =

f (x) + f ((x))
f (x) + f (x)
=
= g(x)
2
2

Tambin calculemos
h(x) =

f (x) f (x)
f (x) f ((x))
=
= h(x)
2
2

por lo tanto, h es una funcin impar y continua.


Ejemplo 4.34. Supngase que g y h son dos funciones continuas en a y g(a) = h(a).
Defnase una funcin f de la manera siguiente

g(x) , x a
f (x) =

h(x) , x a
Probar que f es una funcin continua en a.
Prueba: Sea  > 0 dada arbitrariamente, busquemos el > 0 adecuado, para ello, como
g es continua en a, luego existe 1 > 0 tal que
x R, |x a| < 1 |g(x) h(a)| < 

(4.9)

Tambin por la continuidad de h en el punto a, tenemos garantizada la existencia de un


2 > 0 de modo que
x R, |x a| < 2 |h(x) h(a)| < 

(4.10)

finalmente podemos considerar un radio de manera que se cumpla con las ecuaciones (4.9)
y (??) tomemos
= min{1 , 2 }
Veamos que, para este > 0 cumple con las condiciones pedidas:
Sea x R con |x a| < , entonces tenemos sobre x dos casos a tratar:

104

CAPTULO 4. PROBLEMAS RESUELTOS

(i) Si x < a
para este caso, podemos calcular
|f (x) f (a)| = |h(x) h(a)| < 
(ii) Si a a
Podemos calcular
|f (x) f (a)| = |g(x) g(a)| < 
Finalmente de ambos casos obtenemos que |f (x)f (a)| < , por lo tanto f es una funcin
continua en a.
Ejemplo 4.35. Pruebe que las siguientes afirmaciones son equivalentes:
(a) Si f es una funcin continua en [a, b] y f (a) < 0 < f (b), entonces existe algn
x (a, b) tal que f (x) = 0.
(b) Si f es continua en [a, b] y f (a) < c < f (b), entonces existe x (a, b) tal que
f (x) = c.
Prueba: Recordar que lo que me estn pidiendo demostrar es la bicondicional siguiente
(a) (b), luego la demostracin consta en dos implicaciones, veamos cada una de ellas.
()
Para esta parte tenemos
Hiptesis: Si f es una funcin continua en [a, b] y f (a) < 0 < f (b), entonces existe algn
x (a, b) tal que f (x) = 0.
Tesis: Si f es continua en [a, b] y f (a) < c < f (b), entonces existe x (a, b) tal que
f (x) = c.
Consideremos una nueva funcin
g(x) = f (x) c
es claro que se cumple
(i) g es una funcin continua en [a, b].
(ii) g(a) < 0 < g(b).

105
ahora, por dato del problema, debe existir x (a, b) de modo que g(x) = 0, luego por la
definicin dada, tenemos
f (x) c = 0
es decir f (x) = c, la cual, es justamente lo que me peda el problema.
()
En esta parte tenemos que
Hiptesis: Si f es continua en [a, b] y f (a) < c < f (b), entonces existe x (a, b) tal que
f (x) = c.
Tesis: Si f es una funcin continua en [a, b] y f (a) < 0 < f (b), entonces existe algn
x (a, b) tal que f (x) = 0.
Ahora se tiene que considerar: Si f es una funcin continua en [a, b] y f (a) < 0 < f (b),
pero tenemos por hiptesis que haciendo c = 0 se verifica el antecedente de la hiptesis,
luego podemos concluir que existe x (a, b) de modo que
f (x) = 0
con lo cual, se obtiene lo pedido.
Observacin:
El estudiante debe notar, la importancia de lo que significa una implicacin, con su hiptesis (antecedente) y tesis (consecuente o conclusin), en que momento de la demostracin
utilizarla y cmo. La resolucin anterior es necesario meditarla, por la manera en que
utiliza las hiptesis.
Ejemplo 4.36. Demostrar la equivalencia de las afirmaciones siguientes
a) Si f es continua en [a, b], entonces f est acotada superiormente en [a, b].
b) Si f es continua en [a, b], entonces f est acotada inferiormente en [a, b].
Prueba: tenemos que demostrar por doble implicacin, de la manera siguiente:
(a b)
Debido a que f es una funcin continua en [a, b], luego f es tambin continua sobre el
intervalo compacto [a, b]. Por lo tanto podemos utilizar o aplicar la hiptesis, note que

106

CAPTULO 4. PROBLEMAS RESUELTOS

la hiptesis no est dada nicamente para una funcin f en particular, sino para toda
funcin g continua en [a, b], en esta parte el estudiante debe notar la diferencia de un dato
concreto y el antecedente de una implicacin.
Ahora tenemos que f est acotada superiormente, luego
x [a, b], (f )(x) k
para algn k R, luego f (x) k, por lo tanto
x [a, b], k f (x)
esto significa que f esta acotada inferiormente sobre el intervalo [a, b].
(b a)
Se deja al lector para su resolucin.

Ejemplo 4.37. Probar que todo nmero positivo tiene una raz cuadrada.
Prueba: Sea r R, r > 0 dado en forma arbitraria.
Pruebe que existe b R de modo que r = b2 , otra forma de denotar ello es

r = b.

Considere la siguiente funcin clave para la demostracin


f (x) = x2 r
la cual es una funcin cuadrtica continua en todo su dominio R, podemos evaluar en el
siguiente para de valores
f (0) = r < 0
tambin
1
1
1
f (r + ) = (r + )2 r = r2 + > 0
2
2
4
por lo tanto tenemos la siguiente desigualdad convenientemente tomada
1
f (0) < 0 < f (r + )
2
luego, podemos indicar que f es continua sobre el compacto [0, r + 21 ], por lo tanto existe
b (0, r + 12 ) tal que f b) = 0, es decir b2 r = 0, finalmente
b2 = r

107
Ejemplo 4.38. Muestre que, para cada d (3, 5), existe c (0, 1) de modo que
c8 + c5 c4 + c + 3 = d.
Prueba: Considere d (3, 5) arbitrario, luego formando la siguiente funcin f (x) =
x8 + x5 x4 + x + 3 d la cual es continua en [0, 1], realizando el primer clculo
f (0) = 3 d < 0
Ahora realizamos el segundo clculo
f (1) = 5 d > 0
por lo tanto f (0) < 0 < f (1), podemos aplicar el TVI, con la cual existe c (0, 1) de
modo que f (c) = 0, es decir
c8 + c5 c4 + c + 3 d = 0
y listo ello, era lo que me pedan.

5
Problemas Propuestos
1. Considere la siguiente funcin f (x) =

x+2
, f es continua en el punto x = 3?
x2

2. Encontrar los valores de a y b para que la siguiente funcin sea continua

2x a , x < 2
f (x) =

6x b , x 2.
3. Encuentre los valores de a y b de tal forma que la siguiente funcin

ax + x2 1
,x > 1
f (x) =

ax x3 + 3 , x 1.

109

Você também pode gostar